Как найти угловой коэффициент касательной к графику функции: Найдите угловой коэффициент касательной проведенной к графику функции y=x^4-2x^3+3x-13 в точке x0=-1

Содержание

Уравнение касательной к графику функции. Как его найти?

В чем заключается геометрический смысл производной

Одну из главных ролей в записи касательной к графику играет производная, поэтому определим ее геометрический смысл.

Пусть задана произвольная функция y = f(x).

На графике этой функции возьмем точку А с координатами . А теперь выберем точку B с координатами недалеко от точки А.

Проведем через точки A и B прямую.

Угол наклона прямой к оси абсцисс обозначим буквой .

Проведем через точку А прямую, параллельную оси абсцисс, а через точку B — прямую, параллельную оси ординат. Пусть эти две прямые пересекутся в точке C.

Тогда катет , а катет .

Если взять отношения этих значений , то получим отношение противолежащего катета к прилежащему катету в прямоугольном треугольнике ABC, что равно .

Если уменьшать расстояние между точками A и B, то будут уменьшаться длины отрезков и и в какой-то момент точка В совпадет с точкой A, а отношение станет равно производной функции y = f(x) в точке .

Тут может возникнуть вопрос: при чем здесь геометрический смысл производной, если мы начали с касательной?

Касательная — это прямая. Вспомним уравнение прямой: y = kx + b, где k — это коэффициент наклона прямой, и он равен тангенсу угла между прямой и осью абсцисс. А теперь совмещаем все данные и делаем вывод, что .

Это очень важный для нас вывод, попробуем применить его на практике, а именно на задачах формата профильного ЕГЭ по математике.

Практикующий детский психолог Екатерина Мурашова

Бесплатный курс для современных мам и пап от Екатерины Мурашовой. Запишитесь и участвуйте в розыгрыше 8 уроков

Решение задач

Задача 1

К графику функции y = f(x) проведена касательная в точке с абсциссой . Нужно найти угловой коэффициент касательной к графику данной функции.

Из теории выше мы узнали, как найти угловой коэффициент касательной — он равен тангенсу угла наклона касательной к графику функции в точке. Значит, через целочисленные точки на прямой построим прямоугольный треугольник и найдем отношение противолежащего катета к прилежащему — получится .

Ответ: 3.

Задача 2

К графику функции y = f(x) проведена касательная в точке с абсциссой . Определите угловой коэффициент касательной в точке .

Действуйте по уже известным правилам. Получился ответ 0,25? А вот и нет! В данном случае нужно обратить внимание на убывание графика касательной. Видите, она слева направо идет вниз? Значит, к ответу нужно добавить минус и записать его — получится −0,25.

Ответ: −0,25.

Будьте внимательны

Не позвольте маленькому минусу лишить вас дополнительных баллов на экзамене или контрольной. 😩

Задача 3

На рисунке изображен график функции y = f(x), определенной на интервале (−8; 3). Найдите количество точек, в которых касательная к графику функции параллельна прямой y = 36.

Надеюсь, вы не подумали, что мы будем изображать прямую y = 36 и искать касательные, параллельные ей. 🤯 Достаточно будет рассуждений. Прямая y = 36 — горизонтальная прямая с k = 0, а значит, и у касательных к графику k = 0 или тангенс угла наклона касательной к графику функции также будет равен нулю, что может быть только в точках экстремума функции или, проще говоря, в «бугорках» функции.

В ответе просили указать количество таких точек, значит, ответ — 5.

Ответ: 5.

Задача 4

Прямая y = 4x + 13 параллельна касательной к графику функции . Найдите абсциссу точки касания.

Если прямая параллельна касательной к графику функции, то у них будут равные угловые коэффициенты. Угловой коэффициент прямой y = 4x + 13 равен 4, а угловой коэффициент касательной к графику функции равен производной от этой функции, то есть 2x − 3. Приравняем полученные значения и найдем x:

2x − 3 = 4;

x = 3,5.

Ответ: 3,5 — абсцисса точки касания.

Как составить уравнение касательной к графику функции

Но как поступать, если нужно составить уравнение касательной к графику функции?

Уравнение касательной к графику функции y = f(x) в точке находится по формуле .

Для упрощения понимания этой формулы запишем алгоритм составления уравнения касательной к кривой y = f(x) в точке :

  1. Вычислим значение функции в точке касания, для этого подставим в y = f(x) и посчитаем.

  2. Продифференцируем функцию y = f(x).

  3. Вычислим значение функции в точке касания, для этого подставим в и посчитаем.

  4. Составим уравнение касательной и приведем его к виду y = kx + b.

Задача 5

Запишите уравнение касательной к параболе в точке .

Воспользуемся алгоритмом выше:

  1. Вычислим значение функции в точке касания, для этого подставим в и посчитаем: .

  2. Продифференцируем функцию: .

  3. Вычислим значение функции в точке касания: .

  4. Все найденные значения подставим в уравнение касательной: .

  5. Приведем полученное выражение к виду y = kx + b: y = −2x + 24.

Ответ: уравнение касательной y = −2x + 24.

По условию задачи нас не просили, но мы можем изобразить график квадратичной функции и касательную к параболе для проверки. Если получилась лишь одна точка касания с правильными координатами, значит, наши расчеты были верны!

Некоторые темы математики, как клубок ниток, содержат в себе понятия и правила из других тем. Не понимая прошлые темы, не удастся разобраться и в новой. На каждом уроке курсов обучения математике в онлайн-школе Skysmart мы актуализируем уже имеющиеся знания, поэтому не разобраться не получится.

Приходите на бесплатный вводный урок за подробным разбором сильных и слабых сторон и конкретными рекомендациями, как улучшить оценки и подготовиться к экзаменам!

Формула касательной к графику через производную. Как найти угловой коэффициент

Главная > Идеи > Формула касательной к графику через производную. Как найти угловой коэффициент

Касательная – это прямая, проходящая через точку кривой и совпадающая с ней в этой точке с точностью до первого порядка (рис.1).

Другое определение : это предельное положение секущей при Δx →0.

Пояснение : Возьмем прямую, пересекающую кривую в двух точках: А и b (см.рисунок). Это секущая. Будем поворачивать ее по часовой стрелке до тех пор, пока она не обретет только одну общую точку с кривой. Так мы получим касательную.

Строгое определение касательной:

Касательная к графику функции

f , дифференцируемой в точке x о , — это прямая, проходящая через точку (x о ; f (x о )) и имеющая угловой коэффициент f ′(x о ).

Угловой коэффициент имеет прямая вида y = kx + b . Коэффициент k и является угловым коэффициентом этой прямой.

Угловой коэффициент равен тангенсу острого угла, образуемого этой прямой с осью абсцисс:


k = tg α

Здесь угол α – это угол между прямой y = kx + b и положительным (то есть против часовой стрелки) направлением оси абсцисс. Он называется углом наклона прямой (рис.1 и 2).

Если угол наклона прямой y = kx + b острый, то угловой коэффициент является положительным числом. График возрастает (рис.1).

Если угол наклона прямой y = kx + b тупой, то угловой коэффициент является отрицательным числом. График убывает (рис.2).

Если прямая параллельна оси абсцисс, то угол наклона прямой равен нулю. В этом случае угловой коэффициент прямой тоже равен нулю (так как тангенс нуля есть ноль). Уравнение прямой будет иметь вид y = b (рис.3).

Если угол наклона прямой равен 90º (π/2), то есть она перпендикулярна оси абсцисс, то прямая задается равенством x = c , где c – некоторое действительное число (рис.4).

Уравнение касательной к графику функции y = f (x ) в точке x о :


Пример : Найдем уравнение касательной к графику функции f (x ) = x 3 – 2x 2 + 1 в точке с абсциссой 2.

Решение .

Следуем алгоритму.

1) Точка касания x о равна 2. Вычислим f (x о ):

f (x о ) = f (2) = 2 3 – 2 ∙ 2 2 + 1 = 8 – 8 + 1 = 1

2) Находим f ′(x ). Для этого применяем формулы дифференцирования, изложенные в предыдущем разделе. Согласно этим формулам, х 2 = 2х , а х 3 = 3х 2 . Значит:

f ′(x ) = 3х 2 – 2 ∙ 2х = 3х 2 – 4х .

Теперь, используя полученное значение f ′(x ), вычислим f ′(x о ):

f ′(x о ) = f ′(2) = 3 ∙ 2 2 – 4 ∙ 2 = 12 – 8 = 4.

3) Итак, у нас есть все необходимые данные: x о = 2, f (x о ) = 1, f ′(x о ) = 4. Подставляем эти числа в уравнение касательной и находим окончательное решение:

у = f (x о ) + f ′(x о ) (x – x о ) = 1 + 4 ∙ (х – 2) = 1 + 4х – 8 = –7 + 4х = 4х – 7.

Ответ : у = 4х – 7.

В этой статье мы разберем все типы задач на нахождение

Вспомним геометрический смысл производной : если к графику функции в точке проведена касательная, то коэффициент наклона касательной (равный тангенсу угла между касательной и положительным направлением оси ) равен производной функции в точке .


Возьмем на касательной произвольную точку с координатами :


И рассмотрим прямоугольный треугольник :


В этом треугольнике

Отсюда

Это и есть уравнение касательной, проведенной к графику функции в точке .

Чтобы написать уравнение касательной, нам достаточно знать уравнение функции и точку, в которой проведена касательная. Тогда мы сможем найти и .

Есть три основных типа задач на составление уравнения касательной.

1. Дана точка касания

2. Дан коэффициент наклона касательной, то есть значение производной функции в точке .

3. Даны координаты точки, через которую проведена касательная, но которая не является точкой касания.

Рассмотрим каждый тип задач.

1 . Написать уравнение касательной к графику функции в точке .

.

б) Найдем значение производной в точке . Сначала найдем производную функции

Подставим найденные значения в уравнение касательной:

Раскроем скобки в правой части уравнения. Получим:

Ответ: .

2 . Найти абсциссы точек, в которых касательные к графику функции параллельны оси абсцисс.

Если касательная параллельна оси абсцисс, следовательно угол между касательной и положительным направлением оси равен нулю, следовательно тангенс угла наклона касательной равен нулю.

Значит, значение производной функции в точках касания равно нулю.

а) Найдем производную функции .

б) Приравняем производную к нулю и найдем значения , в которых касательная параллельна оси :

Приравняем каждый множитель к нулю, получим:

Ответ: 0;3;5

3 . Написать уравнения касательных к графику функции , параллельных прямой .

Касательная параллельна прямой . Коэффициент наклона этой прямой равен -1. Так как касательная параллельна этой прямой, следовательно, коэффициент наклона касательной тоже равен -1. То есть мы знаем коэффициент наклона касательной , а, тем самым, значение производной в точке касания .

Это второй тип задач на нахождение уравнения касательной.

Итак, у нас дана функция и значение производной в точке касания.

а) Найдем точки, в которых производная функции равна -1.

Сначала найдем уравнение производной.

Приравняем производную к числу -1. 2}»>. Мы получили под корнем отрицательное число, равенство не верно, и точка не принадлежит графику функции и не является точкой касания.

Это последний тип задач на нахождение уравнения касательной. Первым делом нам нужно найти абсциссу точки касания .

Найдем значение .

Пусть — точка касания. Точка принадлежит касательной к графику функции . Если мы подставим координаты этой точки в уравнение касательной, то получим верное равенство:

.

Значение функции в точке равно .

Найдем значение производной функции в точке .

Сначала найдем производную функции . Это .

Производная в точке равна .

Подставим выражения для и в уравнение касательной. Получим уравнение относительно :

Решим это уравнение.

Сократим числитель и знаменатель дроби на 2:

Приведем правую часть уравнения к общему знаменателю. Получим:

Упростим числитель дроби и умножим обе части на — это выражение строго больше нуля. 2} {8-3x_0>=0} }}{ }»>

Решим первое уравнение.

Решим квадратное уравнение, получим

Второй корень не удовлетворяет условию title=»8-3x_0>=0″>, следовательно, у нас только одна точка касания и её абсцисса равна .

Напишем уравнение касательной к кривой в точке . Для этого подставим значение в уравнение — мы его уже записывали.

Ответ:
.

Вам понадобится

  • — математический справочник;
  • — тетрадь;
  • — простой карандаш;
  • — ручка;
  • — транспортир;
  • — циркуль.

Инструкция

Примите к сведению, что график дифференцируемой функции f(x) в точке х0 не имеет различий с отрезком касательной. Поэтому он достаточно близким к отрезку l, к проходящему через точки (х0; f(х0)) и (х0+Δx; f(x0 + Δx)). Чтобы задать прямую, проходящую через точку А с коэффициентами (х0; f(х0)), укажите ее угловой коэффициент. При этом он равен Δy/Δx секущей касательной (Δх→0) , а также стремится к числу f‘(x0).

Если значений f‘(x0) не существует, то, касательной нет, или же она проходит вертикально. Исходя из этого, производной функции в точке х0 объясняется существованием невертикальной касательной, которая соприкасается с графиком функции в точке (х0, f(х0)). В данном случае угловой коэффициент касательной равняется f»(х0). Становится понятен геометрический производной, то есть углового коэффициента касательной.

То есть для того чтобы найти угловой коэффициент касательной, нужно найти значение производной функции в точке касания. Пример: найти угловой коэффициент касательной к функции у = х³ в точке с абсциссой Х0 = 1. Решение: Найдите производную данной функции у΄(х) = 3х²; найдите значение производной в точке Х0 = 1. у΄(1) = 3 × 1² = 3. Угловой коэффициент касательной в точке Х0 = 3.

Начертите на рисунке дополнительные касательные таким образом, чтобы они соприкасались с графиком функции в точках: x1, х2 и х3. Отметьте углы, которые образуются данными касательными с осью абсцисс (угол отсчитывается в положительном направлении — от оси до касательной прямой). Например, угол α1 будет острым, же (α2) – тупой, ну а третий (α3) будет равняться нулю, так как проведенная касательная прямая является параллельной оси ОХ. В этом случае тангенс тупого угла есть отрицательное значение, а тангенс острого угла – положительное, при tg0 и результат равен нулю.

Касательной к заданной окружности называется прямая линяя, которая имеет только одну общую точку с этой окружностью. Касательная к окружности всегда перпендикулярна его радиусу, проведённому к точке касания. Если две касательные проведены из одной точки, не принадлежащей окружности, то расстояния от этой точки до точек касания всегда будет одинаковым. Касательные к окружностям строятся разными способами, в зависимости от их расположения относительно друг друга.

Инструкция

Построение касательной к одной окружности.
1. Строится окружность радиуса R и берётся A, которую будет проходить касательная.
2. Строится окружность с центром в середине отрезка OA и радиусам равным этого отрезка.
3. Пересечения двух точками касания касательных проведённых через точку A к заданной окружности.

Внешняя касательная к двум окружностям .

2. Проводится окружность радиусом R – r с центром в точке O.
3. К полученной окружности проводится касательная из O1, точка касания обозначена M.
4. Радиус R проходящий через точку M на точку T – точку касания окружности.
5. Через центр O1 малой окружности проводится радиус r параллельно R большой окружности. Радиус r указывает на точку T1 – точку касания малой окружности.
окружностям .

Внутренняя касательная к двум окружностям .
1. Строятся две окружности радиусом R и r.
2. Проводится окружность радиусом R + r с центром в точке O.
3. К полученной окружности проводится касательная из точки O1, точка касания обозначена буквой M.
4. Луч OM пересекает первую окружность в точке T – в точке касания большой окружности.
5. Через центр O1 малой окружности проводится радиус r параллельно лучу OM. Радиус r указывает на точку T1 – точку касания малой окружности.
6. Прямая TT1 – касательная к заданным окружностям .

Источники:

  • внутренняя касательная

Угловой шкаф – идеальный вариант для пустующих углов в квартире. Кроме того, конфигурация угловых шкаф ов придает интерьеру классическую атмосферу. В качестве отделки угловых шкаф ов может быть использован любой материал, который подходит для этой цели.

Вам понадобится

  • ДВП, МДФ, шурупы, гвозди, пильный диск, фриз.

Инструкция

Вырежьте из фанеры или ДВП шаблон шириной 125 мм, длиной 1065 мм. Кромки необходимо запилить под углом 45 градусов. По готовому шаблону определите размеры боковых стенок, а так же место, где будет расположен шкаф .

Крышку соедините с боковыми стенками и треугольными полками. Крепление крышки должно происходить к верхним кромкам боковых стенок при помощи шурупов. Для прочности конструкции дополнительно используют клей. Полки прикрепите к планкам.

Наклоните пильный диск под углом 45 градусов и скосите по направляющей планке переднюю кромку боковых стенок. Неподвижные полки прикрепите к планкам МДФ. Соедините боковые стенки при помощи шурупов. Следите за тем, чтобы не было щелей.

В стене сделайте отметки, между которыми поставьте каркас углового шкаф а. С помощью шурупов прикрепите шкаф к стене. Длина дюбеля должна быть 75 мм.

Из цельной плиты МДФ выпилите лицевую рамку. С помощью дисковой пилы вырежьте в ней проемы, используя линейку. Допилите углы.

Найдите значение абсциссы точки касания, которую обозначаются буквой «а». Если она совпадает с заданной точкой касательной, то «а» будет ее х-координате. Определите значение функции f(a), подставив в уравнение функции величину абсциссы.

Определите первую производную уравнения функции f’(x) и подставьте в него значение точки «а».

Возьмите общее уравнение касательной, которое определяется как y = f(a) = f (a)(x – a), и подставьте в него найденные значения a, f(a), f «(a). В результате будет найдено решение графика и касательной.

Решите задачу иным способом, если заданная точка касательной не совпала с точкой касания. В этом случае необходимо в уравнение касательной вместо цифр подставить «а». После этого вместо букв «х» и «у» подставьте значение координат заданной точки. Решите получившееся уравнение, в котором «а» является неизвестной. Поставьте полученное значение в уравнение касательной.

Составьте уравнение касательной с буквой «а», если в условии задачи задано уравнение функции и уравнение параллельной линии относительно искомой касательной. После этого необходимо производную функции , чтобы координату у точки «а». Подставьте соответствующее значение в уравнение касательной и решите функцию.

При составлении уравнения касательной к графику функции используется понятие «абсцисса точки касания». Данная величина может задаваться изначально в условиях задачи или же ее необходимо определять самостоятельно.

Инструкция

Начертите на листе в клеточку оси координат х и у. Изучите заданное уравнение для графика функции. Если оно является , то достаточно два значения для параметра у при любых х, после чего построить найденные точки на оси координат и соединить их линией. Если же график нелинейный, то составьте таблицу зависимости у от х и подберите как минимум пять точек для построения графика.

Определите значение абсциссы точки касания для случая, когда заданная точка касательной не совпадает с графиком функции. Задаем третий параметр буквой «а».

Запишите уравнение функции f(a). Для этого в исходное уравнение вместо х подставьте а. Найдите производную функции f(x) и f(a). Подставьте необходимые данные в общее уравнение касательной, которое имеет вид: y = f(a) + f «(a)(x – a). В результате получить уравнение, которое из трех неизвестных параметров.

Подставьте в него вместо х и у координаты заданной точки, через которую проходит касательная. После этого найдите решение полученного уравнения для всех а. Если оно является квадратным, то будет два значения абсциссы точки касания. Это , что касательная проходит два раза возле графика функции.

Нарисуйте график заданной функции и , которые заданы по условию задачи. В этом случае необходимо также задать неизвестный параметр а и подставить его в уравнение f(a). Приравняйте производную f(a) к производной уравнения параллельной прямой. Данное выходит из условия параллельности двух . Найдите корни полученного уравнения, которые будут являться абсциссами точки касания.

Прямая y=f(x) будет касательной к изображенному на рисунке графику в точке х0 в том случае, если она проходит через точку с координатами (х0; f(x0)) и обладает угловым коэффициентом f»(x0). Найти такой коэффициент, зная особенности касательной, несложно.

Вам понадобится

  • — математический справочник;
  • — простой карандаш;
  • — тетрадь;
  • — транспортир;
  • — циркуль;
  • — ручка.

Инструкция

Если значения f‘(x0) не существует, то либо касательной нет, либо она проходит вертикально. Ввиду этого, наличие производной функции в точке х0 обусловлено существованием невертикальной касательной, соприкасающейся с графиком функции в точке (х0, f(х0)). В этом случае угловой коэффициент касательной равен будет f»(х0). Таким образом, становится ясен геометрический смысл производной – расчет углового коэффициента касательной.

Определите общую . Подобного рода сведения можно получить, обратившись к данным переписи населения. Для определения общих коэффициентов рождаемости, смертности, брачности и разводимости вам понадобится найти произведение общей населения и расчетного периода. Получившееся число запишите в знаменатель.

Поставьте на числителя показатель, соответствующий искомому относительному. Например, если перед вами стоит определить общий коэффициент рождаемости, то на месте числителя должно находиться число, отражающее общее количество рожденных за интересующий вас период. Если вашей целью является уровня смертности или брачности, то на место числителя поставьте число умерших в расчетный период или число вступивших в брак, соответственно.

Умножьте получившееся число на 1000. Это и будет искомый вами общий коэффициент. Если же перед вами стоит задача найти общий коэффициент прироста, то вычтите из коэффициента рождаемости коэффициент смертности.

Видео по теме

Источники:

  • Общие коэффициенты естественного движения населения

Главным показателем эффективности экстракции является коэффициент распределения . Он считается по формуле: Со/Св, где Со – концентрация извлекаемого вещества в органическом растворителе (экстракторе), а Св – концентрация этого же вещества в воде, после наступления равновесия. Как можно опытным путем найти коэффициент распределения?

Касательная — это прямая , которая касается графика функции в одной точке и все точки которой находятся на наименьшем расстоянии от графика функции. Поэтому касательная проходит касательно графика функции под определённым углом и не могут проходить через точку касания несколько касательных под разными углами. Уравнения касательной и уравнения нормали к графику функции составляются с помощью производной.

Уравнение касательной выводится из уравнения прямой .

Выведем уравнение касательной, а затем — уравнение нормали к графику функции.

y = kx + b .

В нём k — угловой коэффициент.

Отсюда получаем следующую запись:

y y 0 = k (x x 0 ) .

Значение производной f «(x 0 ) функции y = f (x ) в точке x 0 равно угловому коэффициенту k = tgφ касательной к графику функции, проведённой через точку M 0 (x 0 , y 0 ) , где y 0 = f (x 0 ) . В этом состоит геометрический смысл производной .

Таким образом, можем заменить k на f «(x 0 ) и получить следующее уравнение касательной к графику функции :

y y 0 = f «(x 0 )(x x 0 ) .

В задачах на составление уравнения касательной к графику функции (а мы уже скоро к ним перейдём) требуется привести получившееся по вышеприведённой формуле уравнение к уравнению прямой в общем виде . Для этого нужно все буквы и числа перенести в левую часть уравнения, а в правой части оставить ноль.

Теперь об уравнении нормали. Нормаль — это прямая, проходящая через точку касания к графику функции перпендикулярно касательной. Уравнение нормали :

(x x 0 ) + f «(x 0 )(y y 0 ) = 0

Для разминки первый же пример прелагается решить самостоятельно, а затем посмотреть решение. Есть все основания надеяться, что для наших читателей эта задача не будет «холодным душем».

Пример 0. Составить уравнение касательной и уравнение нормали к графику функции в точке M (1, 1) .

Пример 1. Составить уравнение касательной и уравнение нормали к графику функции , если абсцисса точки касания .

Найдём производную функции:

Теперь у нас есть всё, что требуется подставить в приведённую в теоретической справке запись, чтобы получить уравнение касательной. Получаем

В этом примере нам повезло: угловой коэффициент оказался равным нулю, поэтому отдельно приводить уравнение к общему виду не понадобилось. Теперь можем составить и уравнение нормали:

На рисунке ниже: график функции бордового цвета, касательная зелёного цвета, нормаль оранжевого цвета.

Следующий пример — тоже не сложный: функция, как и в предыдущем, также представляет собой многочлен, но угловой коэффициен не будет равен нулю, поэтому добавится ещё один шаг — приведение уравнения к общему виду.

Пример 2.

Решение. Найдём ординату точки касания:

Найдём производную функции:

.

Найдём значение производной в точке касания, то есть угловой коэффициент касательной:

Подставляем все полученные данные в «формулу-болванку» и получаем уравнение касательной:

Приводим уравнение к общему виду (все буквы и числа, отличные от нуля, собираем в левой части, а в правой оставляем ноль):

Составляем уравнение нормали:

Пример 3. Составить уравнение касательной и уравнение нормали к графику функции , если абсцисса точки касания .

Решение. Найдём ординату точки касания:

Найдём производную функции:

.

Найдём значение производной в точке касания, то есть угловой коэффициент касательной:

.

Находим уравнение касательной:

Перед тем, как привести уравнение к общему виду, нужно его немного «причесать»: умножить почленно на 4. Делаем это и приводим уравнение к общему виду:

Составляем уравнение нормали:

Пример 4. Составить уравнение касательной и уравнение нормали к графику функции , если абсцисса точки касания .

Решение. Найдём ординату точки касания:

.

Найдём производную функции:

Найдём значение производной в точке касания, то есть угловой коэффициент касательной:

.

Получаем уравнение касательной:

Приводим уравнение к общему виду:

Составляем уравнение нормали:

Распространённая ошибка при составлении уравнений касательной и нормали — не заметить, что функция, данная в примере, — сложная и вычислять её производную как производную простой функции. Следующие примеры — уже со сложными функциями (соответствующий урок откроется в новом окне).

Пример 5. Составить уравнение касательной и уравнение нормали к графику функции , если абсцисса точки касания .

Решение. Найдём ординату точки касания:

Внимание! Данная функция — сложная, так как аргумент тангенса (2x ) сам является функцией. Поэтому найдём производную функции как производную сложной функции.

С понятием касательной к графику функции вы уже знакомы. График дифференцируемой в точке х 0 функции f вблизи х 0 практически не отличается от отрезка касательной, а значит, он близок к отрезку секущей l, проходящей через точки (х 0 ; f (х 0)) и (х 0 +Δx; f (x 0 + Δx)). Любая из таких секущих проходит через точку А (х 0 ; f (х 0)) графика (рис. 1). Для того чтобы однозначно задать прямую, проходящую через данную точку A, достаточно указать ее угловой коэффициент. Угловой коэффициент Δy/Δx секущей при Δх→0 стремится к числу f ‘(x 0) (его мы примем за угловой коэффициент касательной) Говорят, что касательная есть предельное положение секущей при Δх→0 .

Если же f’(х 0) не существует, то касательная либо не существует (как у функции у = |x| в точке (0; 0), см. рис.), либо вертикальна (как у графика функции в точке (0; 0), рис.2).

Итак, существование производной функции f в точке хо эквивалентно существованию (невертикальной) касательной в точке (х 0 , f (х 0)) графика, при этом угловой коэффициент касательной равен f» (х 0). В этом состоитгеометрический смысл производной

Касательная к графику дифференцируемой в точке xо функции f — это прямая, проходящая через точку (x 0 ; f (x 0)) и имеющая угловой коэффициент f ‘(х 0).

Проведем касательные к графику функции f в точках x 1 , х 2 , х 3 (рис. 3) и отметим углы, которые они образуют с осью абсцисс. (Это угол, отсчитываемый в положительном направлении от положительного направления оси до прямой.) Мы видим, что угол α 1 острый, угол α 3 тупой, а угол α 2 равен нулю, так как прямая l параллельна оси Ох. Тангенс острого угла положителен, тупого — отрицателен, tg 0 = 0. Поэтому

F»(x 1)>0, f’(x 2)=0, f’(x 3)
Построение касательных в отдельных точках позволяет более точно строить эскизы графиков. Так, например, для построения эскиза графика функции синус предварительно находим, что в точках 0; π/2 и π производная синуса равна 1; 0 и -1 соответственно. Построим прямые, проходящие через точки (0; 0), (π/2,1) и (π, 0) с угловыми коэффициентами 1, 0 и -1 соответственно (рис. 4) Остается вписать в полученную трапецию, образованную этими прямыми и прямой Ох, график синуса так, чтобы при х, равном 0, π/2 и π, он касался соответствующих прямых.

Отметим, что график синуса в окрестности нуля практически не отличим от прямой у = х. Пусть, например, масштабы по осям выбраны так, что единице соответствует отрезок в 1см. Имеем sin 0,5 ≈ 0,479425, т. е. |sin 0,5 — 0,5| ≈ 0,02, и в выбранном масштабе это соответствует отрезку длиной 0,2 мм. Поэтому график функции y = sin x в интервале (-0,5; 0,5) будет отклоняться (в вертикальном направлении) от прямой у = х не более чем на 0,2 мм, что примерно соответствует толщине проводимой линии.

Производная уравнение касательной к графику функции. Как найти угловой коэффициент

Теме «Угловой коэффициент касательной как тангенс угла наклона» в аттестационном экзамене отводится сразу несколько заданий. В зависимости от их условия, от выпускника может требоваться как полный ответ, так и краткий. При подготовке к сдаче ЕГЭ по математике ученику обязательно стоит повторить задачи, в которых требуется вычислить угловой коэффициент касательной.

Сделать это вам поможет образовательный портал «Школково». Наши специалисты подготовили и представили теоретический и практический материал максимально доступно. Ознакомившись с ним, выпускники с любым уровнем подготовки смогут успешно решать задачи, связанные с производными, в которых требуется найти тангенс угла наклона касательной.

Основные моменты

Для нахождения правильного и рационального решения подобных заданий в ЕГЭ необходимо вспомнить базовое определение: производная представляет собой скорость изменения функции; она равна тангенсу угла наклона касательной, проведенной к графику функции в определенной точке. Не менее важно выполнить чертеж. Он позволит найти правильное решение задач ЕГЭ на производную, в которых требуется вычислить тангенс угла наклона касательной. Для наглядности лучше всего выполнить построение графика на плоскости ОХY.

Если вы уже ознакомились с базовым материалом на тему производной и готовы приступить к решению задач на вычисление тангенса угла наклона касательной, подобных заданиям ЕГЭ, сделать это можно в режиме онлайн. Для каждого задания, например, задач на тему «Связь производной со скоростью и ускорением тела» , мы прописали правильный ответ и алгоритм решения. При этом учащиеся могут попрактиковаться в выполнении задач различного уровня сложности. В случае необходимости упражнение можно сохранить в разделе «Избранное», чтобы потом обсудить решение с преподавателем.

Соблюдение Вашей конфиденциальности важно для нас. По этой причине, мы разработали Политику Конфиденциальности, которая описывает, как мы используем и храним Вашу информацию. Пожалуйста, ознакомьтесь с нашими правилами соблюдения конфиденциальности и сообщите нам, если у вас возникнут какие-либо вопросы.

Сбор и использование персональной информации

Под персональной информацией понимаются данные, которые могут быть использованы для идентификации определенного лица либо связи с ним.

От вас может быть запрошено предоставление вашей персональной информации в любой момент, когда вы связываетесь с нами.

Ниже приведены некоторые примеры типов персональной информации, которую мы можем собирать, и как мы можем использовать такую информацию.

Какую персональную информацию мы собираем:

  • Когда вы оставляете заявку на сайте, мы можем собирать различную информацию, включая ваши имя, номер телефона, адрес электронной почты и т.д.

Как мы используем вашу персональную информацию:

  • Собираемая нами персональная информация позволяет нам связываться с вами и сообщать об уникальных предложениях, акциях и других мероприятиях и ближайших событиях.
  • Время от времени, мы можем использовать вашу персональную информацию для отправки важных уведомлений и сообщений.
  • Мы также можем использовать персональную информацию для внутренних целей, таких как проведения аудита, анализа данных и различных исследований в целях улучшения услуг предоставляемых нами и предоставления Вам рекомендаций относительно наших услуг.
  • Если вы принимаете участие в розыгрыше призов, конкурсе или сходном стимулирующем мероприятии, мы можем использовать предоставляемую вами информацию для управления такими программами.

Раскрытие информации третьим лицам

Мы не раскрываем полученную от Вас информацию третьим лицам.

Исключения:

  • В случае если необходимо — в соответствии с законом, судебным порядком, в судебном разбирательстве, и/или на основании публичных запросов или запросов от государственных органов на территории РФ — раскрыть вашу персональную информацию. Мы также можем раскрывать информацию о вас если мы определим, что такое раскрытие необходимо или уместно в целях безопасности, поддержания правопорядка, или иных общественно важных случаях.
  • В случае реорганизации, слияния или продажи мы можем передать собираемую нами персональную информацию соответствующему третьему лицу – правопреемнику.

Защита персональной информации

Мы предпринимаем меры предосторожности — включая административные, технические и физические — для защиты вашей персональной информации от утраты, кражи, и недобросовестного использования, а также от несанкционированного доступа, раскрытия, изменения и уничтожения.

Соблюдение вашей конфиденциальности на уровне компании

Для того чтобы убедиться, что ваша персональная информация находится в безопасности, мы доводим нормы соблюдения конфиденциальности и безопасности до наших сотрудников, и строго следим за исполнением мер соблюдения конфиденциальности.

С понятием касательной к графику функции вы уже знакомы. График дифференцируемой в точке х 0 функции f вблизи х 0 практически не отличается от отрезка касательной, а значит, он близок к отрезку секущей l, проходящей через точки (х 0 ; f (х 0)) и (х 0 +Δx; f (x 0 + Δx)). Любая из таких секущих проходит через точку А (х 0 ; f (х 0)) графика (рис. 1). Для того чтобы однозначно задать прямую, проходящую через данную точку A, достаточно указать ее угловой коэффициент. Угловой коэффициент Δy/Δx секущей при Δх→0 стремится к числу f ‘(x 0) (его мы примем за угловой коэффициент касательной) Говорят, что касательная есть предельное положение секущей при Δх→0 .

Если же f’(х 0) не существует, то касательная либо не существует (как у функции у = |x| в точке (0; 0), см. рис.), либо вертикальна (как у графика функции в точке (0; 0), рис.2).

Итак, существование производной функции f в точке хо эквивалентно существованию (невертикальной) касательной в точке (х 0 , f (х 0)) графика, при этом угловой коэффициент касательной равен f» (х 0). В этом состоитгеометрический смысл производной

Касательная к графику дифференцируемой в точке xо функции f — это прямая, проходящая через точку (x 0 ; f (x 0)) и имеющая угловой коэффициент f ‘(х 0).

Проведем касательные к графику функции f в точках x 1 , х 2 , х 3 (рис. 3) и отметим углы, которые они образуют с осью абсцисс. (Это угол, отсчитываемый в положительном направлении от положительного направления оси до прямой.) Мы видим, что угол α 1 острый, угол α 3 тупой, а угол α 2 равен нулю, так как прямая l параллельна оси Ох. Тангенс острого угла положителен, тупого — отрицателен, tg 0 = 0. Поэтому

F»(x 1)>0, f’(x 2)=0, f’(x 3)
Построение касательных в отдельных точках позволяет более точно строить эскизы графиков. Так, например, для построения эскиза графика функции синус предварительно находим, что в точках 0; π/2 и π производная синуса равна 1; 0 и -1 соответственно. Построим прямые, проходящие через точки (0; 0), (π/2,1) и (π, 0) с угловыми коэффициентами 1, 0 и -1 соответственно (рис. 4) Остается вписать в полученную трапецию, образованную этими прямыми и прямой Ох, график синуса так, чтобы при х, равном 0, π/2 и π, он касался соответствующих прямых.

Отметим, что график синуса в окрестности нуля практически не отличим от прямой у = х. Пусть, например, масштабы по осям выбраны так, что единице соответствует отрезок в 1см. Имеем sin 0,5 ≈ 0,479425, т. е. |sin 0,5 — 0,5| ≈ 0,02, и в выбранном масштабе это соответствует отрезку длиной 0,2 мм. Поэтому график функции y = sin x в интервале (-0,5; 0,5) будет отклоняться (в вертикальном направлении) от прямой у = х не более чем на 0,2 мм, что примерно соответствует толщине проводимой линии.

Пусть дана функция f , которая в некоторой точке x 0 имеет конечную производную f (x 0). Тогда прямая, проходящая через точку (x 0 ; f (x 0)), имеющая угловой коэффициент f ’(x 0), называется касательной.

А что будет, если производная в точке x 0 не существует? Возможны два варианта:

  1. Касательная к графику тоже не существует. Классический пример — функция y = |x | в точке (0; 0).
  2. Касательная становится вертикальной. Это верно, к примеру, для функции y = arcsin x в точке (1; π /2).

Уравнение касательной

Всякая невертикальная прямая задается уравнением вида y = kx + b , где k — угловой коэффициент. Касательная — не исключение, и чтобы составить ее уравнение в некоторой точке x 0 , достаточно знать значение функции и производной в этой точке.

Итак, пусть дана функция y = f (x ), которая имеет производную y = f ’(x ) на отрезке . Тогда в любой точке x 0 ∈ (a ; b ) к графику этой функции можно провести касательную, которая задается уравнением:

y = f ’(x 0) · (x − x 0) + f (x 0)

Здесь f ’(x 0) — значение производной в точке x 0 , а f (x 0) — значение самой функции.

Задача. Дана функция y = x 3 . Составить уравнение касательной к графику этой функции в точке x 0 = 2.

Уравнение касательной: y = f ’(x 0) · (x − x 0) + f (x 0). Точка x 0 = 2 нам дана, а вот значения f (x 0) и f ’(x 0) придется вычислять.

Для начала найдем значение функции. Тут все легко: f (x 0) = f (2) = 2 3 = 8;
Теперь найдем производную: f ’(x ) = (x 3)’ = 3x 2 ;
Подставляем в производную x 0 = 2: f ’(x 0) = f ’(2) = 3 · 2 2 = 12;
Итого получаем: y = 12 · (x − 2) + 8 = 12x − 24 + 8 = 12x − 16.
Это и есть уравнение касательной.

Задача. Составить уравнение касательной к графику функции f (x ) = 2sin x + 5 в точке x 0 = π /2.

В этот раз не будем подробно расписывать каждое действие — укажем лишь ключевые шаги. Имеем:

f (x 0) = f (π /2) = 2sin (π /2) + 5 = 2 + 5 = 7;
f ’(x ) = (2sin x + 5)’ = 2cos x ;
f ’(x 0) = f ’(π /2) = 2cos (π /2) = 0;

Уравнение касательной:

y = 0 · (x − π /2) + 7 ⇒ y = 7

В последнем случае прямая оказалась горизонтальной, т.к. ее угловой коэффициент k = 0. Ничего страшного в этом нет — просто мы наткнулись на точку экстремума.

В этой статье мы разберем все типы задач на нахождение

Вспомним геометрический смысл производной : если к графику функции в точке проведена касательная, то коэффициент наклона касательной (равный тангенсу угла между касательной и положительным направлением оси ) равен производной функции в точке .


Возьмем на касательной произвольную точку с координатами :


И рассмотрим прямоугольный треугольник :


В этом треугольнике

Отсюда

Это и есть уравнение касательной, проведенной к графику функции в точке .

Чтобы написать уравнение касательной, нам достаточно знать уравнение функции и точку, в которой проведена касательная. Тогда мы сможем найти и .

Есть три основных типа задач на составление уравнения касательной.

1. Дана точка касания

2. Дан коэффициент наклона касательной, то есть значение производной функции в точке .

3. Даны координаты точки, через которую проведена касательная, но которая не является точкой касания.

Рассмотрим каждый тип задач.

1 . Написать уравнение касательной к графику функции в точке .

.

б) Найдем значение производной в точке . Сначала найдем производную функции

Подставим найденные значения в уравнение касательной:

Раскроем скобки в правой части уравнения. Получим:

Ответ: .

2 . Найти абсциссы точек, в которых касательные к графику функции параллельны оси абсцисс.

Если касательная параллельна оси абсцисс, следовательно угол между касательной и положительным направлением оси равен нулю, следовательно тангенс угла наклона касательной равен нулю. Значит, значение производной функции в точках касания равно нулю.

а) Найдем производную функции .

б) Приравняем производную к нулю и найдем значения , в которых касательная параллельна оси :

Приравняем каждый множитель к нулю, получим:

Ответ: 0;3;5

3 . Написать уравнения касательных к графику функции , параллельных прямой .

Касательная параллельна прямой . Коэффициент наклона этой прямой равен -1. Так как касательная параллельна этой прямой, следовательно, коэффициент наклона касательной тоже равен -1. То есть мы знаем коэффициент наклона касательной , а, тем самым, значение производной в точке касания .

Это второй тип задач на нахождение уравнения касательной.

Итак, у нас дана функция и значение производной в точке касания.

а) Найдем точки, в которых производная функции равна -1.

Сначала найдем уравнение производной.

Приравняем производную к числу -1. 2}»>. Мы получили под корнем отрицательное число, равенство не верно, и точка не принадлежит графику функции и не является точкой касания.

Это последний тип задач на нахождение уравнения касательной. Первым делом нам нужно найти абсциссу точки касания .

Найдем значение .

Пусть — точка касания. Точка принадлежит касательной к графику функции . Если мы подставим координаты этой точки в уравнение касательной, то получим верное равенство:

.

Значение функции в точке равно .

Найдем значение производной функции в точке .

Сначала найдем производную функции . Это .

Производная в точке равна .

Подставим выражения для и в уравнение касательной. Получим уравнение относительно :

Решим это уравнение.

Сократим числитель и знаменатель дроби на 2:

Приведем правую часть уравнения к общему знаменателю. Получим:

Упростим числитель дроби и умножим обе части на — это выражение строго больше нуля. 2} {8-3x_0>=0} }}{ }»>

Решим первое уравнение.

Решим квадратное уравнение, получим

Второй корень не удовлетворяет условию title=»8-3x_0>=0″>, следовательно, у нас только одна точка касания и её абсцисса равна .

Напишем уравнение касательной к кривой в точке . Для этого подставим значение в уравнение — мы его уже записывали.

Ответ:
.

Угловой коэффициент касательной к графику функции положительный. Угловой коэффициент касательной как тангенс угла наклона

Научитесь брать производные от функций. Производная характеризует скорость изменения функции в определенной точке, лежащей на графике этой функции. В данном случае графиком может быть как прямая, так и кривая линия. То есть производная характеризует скорость изменения функции в конкретный момент времени. Вспомните общие правила, по которым берутся производные, и только потом переходите к следующему шагу.

  • Прочитайте статью .
  • Как брать простейшие производные, например, производную показательного уравнения, описано . Вычисления, представленные в следующих шагах, будут основаны на описанных в ней методах.

Научитесь различать задачи, в которых угловой коэффициент требуется вычислить через производную функции. В задачах не всегда предлагается найти угловой коэффициент или производную функции. Например, вас могут попросить найти скорость изменения функции в точке А(х,у). Также вас могут попросить найти угловой коэффициент касательной в точке А(х,у). В обоих случаях необходимо брать производную функции.

  • Возьмите производную данной вам функции. Здесь строить график не нужно – вам понадобится только уравнение функции. В нашем примере возьмите производную функции . Берите производную согласно методам, изложенным в упомянутой выше статье:

    • Производная:
  • В найденную производную подставьте координаты данной вам точки, чтобы вычислить угловой коэффициент. {2}+6x} в точке А(4,2) равен 22.

  • Если возможно, проверьте полученный ответ на графике. Помните, что угловой коэффициент можно вычислить не в каждой точке. Дифференциальное исчисление рассматривает сложные функции и сложные графики, где угловой коэффициент можно вычислить не в каждой точке, а в некоторых случаях точки вообще не лежат на графиках. Если возможно, используйте графический калькулятор, чтобы проверить правильность вычисления углового коэффициента данной вам функции. В противном случае проведите касательную к графику в данной вам точке и подумайте, соответствует ли найденное вами значение углового коэффициента тому, что вы видите на графике.

    • Касательная будет иметь тот же угловой коэффициент, что и график функции в определенной точке. Для того, чтобы провести касательную в данной точке, двигайтесь вправо/влево по оси Х (в нашем примере на 22 значения вправо), а затем вверх на единицу по оси Y. Отметьте точку, а затем соедините ее с данной вам точкой. В нашем примере соедините точки с координатами (4,2) и (26,3).
  • В математике одним из параметров, описывающих положение прямой на декартовой плоскости координат, является угловой коэффициент этой прямой. Этот параметр характеризует наклон прямой к оси абцисс. Чтобы понять, как найти угловой коэффициент, сначала вспомним общий вид уравнения прямой в системе координат XY.

    В общем виде любую прямую можно представить выражением ax+by=c, где a, b и c — произвольные действительные числа, но обязательно a 2 + b 2 ≠ 0.

    Подобное уравнение с помощью несложных преобразований можно довести до вида y=kx+d, в котором k и d — действительные числа. Число k является угловым коэффициентом, а само уравнение прямой подобного вида называется уравнением с угловым коэффициентом. Получается, что для нахождения углового коэффициента, необходимо просто привести исходное уравнение к указанному выше виду. Для более полного понимания рассмотрим конкретный пример:

    Задача: Найти угловой коэффициент линии, заданной уравнением 36x — 18y = 108

    Решение: Преобразуем исходное уравнение.

    Ответ: Искомый угловой коэффициент данной прямой равен 2.

    В случае, если в ходе преобразований уравнения мы получили выражение типа x = const и не можем в результате представить y в виде функции x, то мы имеем дело с прямой, параллельной оси Х. Угловой коэффициент подобной прямой равен бесконечности.

    Для прямых, которых выражены уравнением типа y = const, угловой коэффициент равняется нулю. Это характерно для прямых, параллельных оси абцисс. Например:

    Задача: Найти угловой коэффициент линии, заданной уравнением 24x + 12y — 4(3y + 7) = 4

    Решение: Приведем исходное уравнение к общему виду

    24x + 12y — 12y + 28 = 4

    Из полученного выражения выразить y невозможно, следовательно угловой коэффициент данной прямой равен бесконечности, а сама прямая будет параллельна оси Y.

    Геометрический смысл

    Для лучшего понимания обратимся к картинке:

    На рисунке мы видим график функции типа y = kx. Для упрощения примем коэффициент с = 0. В треугольнике ОАВ отношение стороны ВА к АО будет равно угловому коэффициенту k. Вместе с тем отношение ВА/АО — это тангенс острого угла α в прямоугольном треугольнике ОАВ. Получается, что угловой коэффициент прямой равняется тангенсу угла, который составляет эта прямая с осью абцисс координатной сетки.

    Решая задачу, как найти угловой коэффициент прямой, мы находим тангенс угла между ней и осью Х сетки координат. Граничные случаи, когда рассматриваемая прямая параллельна осям координат, подтверждают вышенаписанное. Действительно для прямой, описанной уравнением y=const, угол между ней и осью абцисс равен нулю. Тангенс нулевого угла также равен нулю и угловой коэффициент тоже равен нулю.

    Для прямых, перпендикулярных оси абцисс и описываемых уравнением х=const, угол между ними и осью Х равен 90 градусов. Тангенс прямого угла равен бесконечности, так же и угловой коэффициент подобных прямых равен бесконечности, что подтверждает написанное выше.

    Угловой коэффициент касательной

    Распространенной, часто встречающейся на практике, задачей является также нахождение углового коэффициента касательной к графику функции в некоторой точке. Касательная — это прямая, следовательно к ней также применимо понятие углового коэффициента.

    Чтобы разобраться, как найти угловой коэффициент касательной, нам будет необходимо вспомнить понятие производной. Производная от любой функции в некоторой точке — это константа, численно равная тангенсу угла, который образуется между касательной в указанной точке к графику этой функции и осью абцисс. Получается, что для определения углового коэффициента касательной в точке x 0 , нам необходимо рассчитать значение производной исходной функции в этой точке k = f»(x 0). Рассмотрим на примере:

    Задача: Найти угловой коэффициент линии, касательной к функции y = 12x 2 + 2xe x при х = 0,1.

    Решение: Найдем производную от исходной функции в общем виде

    y»(0,1) = 24 . 0,1 + 2 . 0,1 . e 0,1 + 2 . e 0,1

    Ответ: Искомый угловой коэффициент в точке х = 0,1 равен 4,831

    Теме «Угловой коэффициент касательной как тангенс угла наклона» в аттестационном экзамене отводится сразу несколько заданий. В зависимости от их условия, от выпускника может требоваться как полный ответ, так и краткий. При подготовке к сдаче ЕГЭ по математике ученику обязательно стоит повторить задачи, в которых требуется вычислить угловой коэффициент касательной.

    Сделать это вам поможет образовательный портал «Школково». Наши специалисты подготовили и представили теоретический и практический материал максимально доступно. Ознакомившись с ним, выпускники с любым уровнем подготовки смогут успешно решать задачи, связанные с производными, в которых требуется найти тангенс угла наклона касательной.

    Основные моменты

    Для нахождения правильного и рационального решения подобных заданий в ЕГЭ необходимо вспомнить базовое определение: производная представляет собой скорость изменения функции; она равна тангенсу угла наклона касательной, проведенной к графику функции в определенной точке. Не менее важно выполнить чертеж. Он позволит найти правильное решение задач ЕГЭ на производную, в которых требуется вычислить тангенс угла наклона касательной. Для наглядности лучше всего выполнить построение графика на плоскости ОХY.

    Если вы уже ознакомились с базовым материалом на тему производной и готовы приступить к решению задач на вычисление тангенса угла наклона касательной, подобных заданиям ЕГЭ, сделать это можно в режиме онлайн. Для каждого задания, например, задач на тему «Связь производной со скоростью и ускорением тела» , мы прописали правильный ответ и алгоритм решения. При этом учащиеся могут попрактиковаться в выполнении задач различного уровня сложности. В случае необходимости упражнение можно сохранить в разделе «Избранное», чтобы потом обсудить решение с преподавателем.

    Вам понадобится

    • — математический справочник;
    • — тетрадь;
    • — простой карандаш;
    • — ручка;
    • — транспортир;
    • — циркуль.

    Инструкция

    Примите к сведению, что график дифференцируемой функции f(x) в точке х0 не имеет различий с отрезком касательной. Поэтому он достаточно близким к отрезку l, к проходящему через точки (х0; f(х0)) и (х0+Δx; f(x0 + Δx)). Чтобы задать прямую, проходящую через точку А с коэффициентами (х0; f(х0)), укажите ее угловой коэффициент. При этом он равен Δy/Δx секущей касательной (Δх→0) , а также стремится к числу f‘(x0).

    Если значений f‘(x0) не существует, то, касательной нет, или же она проходит вертикально. Исходя из этого, производной функции в точке х0 объясняется существованием невертикальной касательной, которая соприкасается с графиком функции в точке (х0, f(х0)). В данном случае угловой коэффициент касательной равняется f»(х0). Становится понятен геометрический производной, то есть углового коэффициента касательной.

    То есть для того чтобы найти угловой коэффициент касательной, нужно найти значение производной функции в точке касания. Пример: найти угловой коэффициент касательной к функции у = х³ в точке с абсциссой Х0 = 1. Решение: Найдите производную данной функции у΄(х) = 3х²; найдите значение производной в точке Х0 = 1. у΄(1) = 3 × 1² = 3. Угловой коэффициент касательной в точке Х0 = 3.

    Начертите на рисунке дополнительные касательные таким образом, чтобы они соприкасались с графиком функции в точках: x1, х2 и х3. Отметьте углы, которые образуются данными касательными с осью абсцисс (угол отсчитывается в положительном направлении — от оси до касательной прямой). Например, угол α1 будет острым, же (α2) – тупой, ну а третий (α3) будет равняться нулю, так как проведенная касательная прямая является параллельной оси ОХ. В этом случае тангенс тупого угла есть отрицательное значение, а тангенс острого угла – положительное, при tg0 и результат равен нулю.

    Касательной к заданной окружности называется прямая линяя, которая имеет только одну общую точку с этой окружностью. Касательная к окружности всегда перпендикулярна его радиусу, проведённому к точке касания. Если две касательные проведены из одной точки, не принадлежащей окружности, то расстояния от этой точки до точек касания всегда будет одинаковым. Касательные к окружностям строятся разными способами, в зависимости от их расположения относительно друг друга.

    Инструкция

    Построение касательной к одной окружности.
    1. Строится окружность радиуса R и берётся A, которую будет проходить касательная.
    2. Строится окружность с центром в середине отрезка OA и радиусам равным этого отрезка.
    3. Пересечения двух точками касания касательных проведённых через точку A к заданной окружности.

    Внешняя касательная к двум окружностям .

    2. Проводится окружность радиусом R – r с центром в точке O.
    3. К полученной окружности проводится касательная из O1, точка касания обозначена M.
    4. Радиус R проходящий через точку M на точку T – точку касания окружности.
    5. Через центр O1 малой окружности проводится радиус r параллельно R большой окружности. Радиус r указывает на точку T1 – точку касания малой окружности.
    окружностям .

    Внутренняя касательная к двум окружностям .
    1. Строятся две окружности радиусом R и r.
    2. Проводится окружность радиусом R + r с центром в точке O.
    3. К полученной окружности проводится касательная из точки O1, точка касания обозначена буквой M.
    4. Луч OM пересекает первую окружность в точке T – в точке касания большой окружности.
    5. Через центр O1 малой окружности проводится радиус r параллельно лучу OM. Радиус r указывает на точку T1 – точку касания малой окружности.
    6. Прямая TT1 – касательная к заданным окружностям .

    Источники:

    • внутренняя касательная

    Угловой шкаф – идеальный вариант для пустующих углов в квартире. Кроме того, конфигурация угловых шкаф ов придает интерьеру классическую атмосферу. В качестве отделки угловых шкаф ов может быть использован любой материал, который подходит для этой цели.

    Вам понадобится

    • ДВП, МДФ, шурупы, гвозди, пильный диск, фриз.

    Инструкция

    Вырежьте из фанеры или ДВП шаблон шириной 125 мм, длиной 1065 мм. Кромки необходимо запилить под углом 45 градусов. По готовому шаблону определите размеры боковых стенок, а так же место, где будет расположен шкаф .

    Крышку соедините с боковыми стенками и треугольными полками. Крепление крышки должно происходить к верхним кромкам боковых стенок при помощи шурупов. Для прочности конструкции дополнительно используют клей. Полки прикрепите к планкам.

    Наклоните пильный диск под углом 45 градусов и скосите по направляющей планке переднюю кромку боковых стенок. Неподвижные полки прикрепите к планкам МДФ. Соедините боковые стенки при помощи шурупов. Следите за тем, чтобы не было щелей.

    В стене сделайте отметки, между которыми поставьте каркас углового шкаф а. С помощью шурупов прикрепите шкаф к стене. Длина дюбеля должна быть 75 мм.

    Из цельной плиты МДФ выпилите лицевую рамку. С помощью дисковой пилы вырежьте в ней проемы, используя линейку. Допилите углы.

    Найдите значение абсциссы точки касания, которую обозначаются буквой «а». Если она совпадает с заданной точкой касательной, то «а» будет ее х-координате. Определите значение функции f(a), подставив в уравнение функции величину абсциссы.

    Определите первую производную уравнения функции f’(x) и подставьте в него значение точки «а».

    Возьмите общее уравнение касательной, которое определяется как y = f(a) = f (a)(x – a), и подставьте в него найденные значения a, f(a), f «(a). В результате будет найдено решение графика и касательной.

    Решите задачу иным способом, если заданная точка касательной не совпала с точкой касания. В этом случае необходимо в уравнение касательной вместо цифр подставить «а». После этого вместо букв «х» и «у» подставьте значение координат заданной точки. Решите получившееся уравнение, в котором «а» является неизвестной. Поставьте полученное значение в уравнение касательной.

    Составьте уравнение касательной с буквой «а», если в условии задачи задано уравнение функции и уравнение параллельной линии относительно искомой касательной. После этого необходимо производную функции , чтобы координату у точки «а». Подставьте соответствующее значение в уравнение касательной и решите функцию.

    При составлении уравнения касательной к графику функции используется понятие «абсцисса точки касания». Данная величина может задаваться изначально в условиях задачи или же ее необходимо определять самостоятельно.

    Инструкция

    Начертите на листе в клеточку оси координат х и у. Изучите заданное уравнение для графика функции. Если оно является , то достаточно два значения для параметра у при любых х, после чего построить найденные точки на оси координат и соединить их линией. Если же график нелинейный, то составьте таблицу зависимости у от х и подберите как минимум пять точек для построения графика.

    Определите значение абсциссы точки касания для случая, когда заданная точка касательной не совпадает с графиком функции. Задаем третий параметр буквой «а».

    Запишите уравнение функции f(a). Для этого в исходное уравнение вместо х подставьте а. Найдите производную функции f(x) и f(a). Подставьте необходимые данные в общее уравнение касательной, которое имеет вид: y = f(a) + f «(a)(x – a). В результате получить уравнение, которое из трех неизвестных параметров.

    Подставьте в него вместо х и у координаты заданной точки, через которую проходит касательная. После этого найдите решение полученного уравнения для всех а. Если оно является квадратным, то будет два значения абсциссы точки касания. Это , что касательная проходит два раза возле графика функции.

    Нарисуйте график заданной функции и , которые заданы по условию задачи. В этом случае необходимо также задать неизвестный параметр а и подставить его в уравнение f(a). Приравняйте производную f(a) к производной уравнения параллельной прямой. Данное выходит из условия параллельности двух . Найдите корни полученного уравнения, которые будут являться абсциссами точки касания.

    Прямая y=f(x) будет касательной к изображенному на рисунке графику в точке х0 в том случае, если она проходит через точку с координатами (х0; f(x0)) и обладает угловым коэффициентом f»(x0). Найти такой коэффициент, зная особенности касательной, несложно.

    Вам понадобится

    • — математический справочник;
    • — простой карандаш;
    • — тетрадь;
    • — транспортир;
    • — циркуль;
    • — ручка.

    Инструкция

    Если значения f‘(x0) не существует, то либо касательной нет, либо она проходит вертикально. Ввиду этого, наличие производной функции в точке х0 обусловлено существованием невертикальной касательной, соприкасающейся с графиком функции в точке (х0, f(х0)). В этом случае угловой коэффициент касательной равен будет f»(х0). Таким образом, становится ясен геометрический смысл производной – расчет углового коэффициента касательной.

    Определите общую . Подобного рода сведения можно получить, обратившись к данным переписи населения. Для определения общих коэффициентов рождаемости, смертности, брачности и разводимости вам понадобится найти произведение общей населения и расчетного периода. Получившееся число запишите в знаменатель.

    Поставьте на числителя показатель, соответствующий искомому относительному. Например, если перед вами стоит определить общий коэффициент рождаемости, то на месте числителя должно находиться число, отражающее общее количество рожденных за интересующий вас период. Если вашей целью является уровня смертности или брачности, то на место числителя поставьте число умерших в расчетный период или число вступивших в брак, соответственно.

    Умножьте получившееся число на 1000. Это и будет искомый вами общий коэффициент. Если же перед вами стоит задача найти общий коэффициент прироста, то вычтите из коэффициента рождаемости коэффициент смертности.

    Видео по теме

    Источники:

    • Общие коэффициенты естественного движения населения

    Главным показателем эффективности экстракции является коэффициент распределения . Он считается по формуле: Со/Св, где Со – концентрация извлекаемого вещества в органическом растворителе (экстракторе), а Св – концентрация этого же вещества в воде, после наступления равновесия. Как можно опытным путем найти коэффициент распределения?

    Соблюдение Вашей конфиденциальности важно для нас. По этой причине, мы разработали Политику Конфиденциальности, которая описывает, как мы используем и храним Вашу информацию. Пожалуйста, ознакомьтесь с нашими правилами соблюдения конфиденциальности и сообщите нам, если у вас возникнут какие-либо вопросы.

    Сбор и использование персональной информации

    Под персональной информацией понимаются данные, которые могут быть использованы для идентификации определенного лица либо связи с ним.

    От вас может быть запрошено предоставление вашей персональной информации в любой момент, когда вы связываетесь с нами.

    Ниже приведены некоторые примеры типов персональной информации, которую мы можем собирать, и как мы можем использовать такую информацию.

    Какую персональную информацию мы собираем:

    • Когда вы оставляете заявку на сайте, мы можем собирать различную информацию, включая ваши имя, номер телефона, адрес электронной почты и т.д.

    Как мы используем вашу персональную информацию:

    • Собираемая нами персональная информация позволяет нам связываться с вами и сообщать об уникальных предложениях, акциях и других мероприятиях и ближайших событиях.
    • Время от времени, мы можем использовать вашу персональную информацию для отправки важных уведомлений и сообщений.
    • Мы также можем использовать персональную информацию для внутренних целей, таких как проведения аудита, анализа данных и различных исследований в целях улучшения услуг предоставляемых нами и предоставления Вам рекомендаций относительно наших услуг.
    • Если вы принимаете участие в розыгрыше призов, конкурсе или сходном стимулирующем мероприятии, мы можем использовать предоставляемую вами информацию для управления такими программами.

    Раскрытие информации третьим лицам

    Мы не раскрываем полученную от Вас информацию третьим лицам.

    Исключения:

    • В случае если необходимо — в соответствии с законом, судебным порядком, в судебном разбирательстве, и/или на основании публичных запросов или запросов от государственных органов на территории РФ — раскрыть вашу персональную информацию. Мы также можем раскрывать информацию о вас если мы определим, что такое раскрытие необходимо или уместно в целях безопасности, поддержания правопорядка, или иных общественно важных случаях.
    • В случае реорганизации, слияния или продажи мы можем передать собираемую нами персональную информацию соответствующему третьему лицу – правопреемнику.

    Защита персональной информации

    Мы предпринимаем меры предосторожности — включая административные, технические и физические — для защиты вашей персональной информации от утраты, кражи, и недобросовестного использования, а также от несанкционированного доступа, раскрытия, изменения и уничтожения.

    Соблюдение вашей конфиденциальности на уровне компании

    Для того чтобы убедиться, что ваша персональная информация находится в безопасности, мы доводим нормы соблюдения конфиденциальности и безопасности до наших сотрудников, и строго следим за исполнением мер соблюдения конфиденциальности.

    Как найти угловой коэффициент касательной. Угловой коэффициент касательной как тангенс угла наклона

    Касательная – это прямая, проходящая через точку кривой и совпадающая с ней в этой точке с точностью до первого порядка (рис.1).

    Другое определение : это предельное положение секущей при Δx →0.

    Пояснение : Возьмем прямую, пересекающую кривую в двух точках: А и b (см.рисунок). Это секущая. Будем поворачивать ее по часовой стрелке до тех пор, пока она не обретет только одну общую точку с кривой. Так мы получим касательную.

    Строгое определение касательной:

    Касательная к графику функции f , дифференцируемой в точке x о , — это прямая, проходящая через точку (x о ; f (x о )) и имеющая угловой коэффициент f ′(x о ).

    Угловой коэффициент имеет прямая вида y = kx + b . Коэффициент k и является угловым коэффициентом этой прямой.

    Угловой коэффициент равен тангенсу острого угла, образуемого этой прямой с осью абсцисс:


    k = tg α

    Здесь угол α – это угол между прямой y = kx + b и положительным (то есть против часовой стрелки) направлением оси абсцисс. Он называется углом наклона прямой (рис.1 и 2).

    Если угол наклона прямой y = kx + b острый, то угловой коэффициент является положительным числом. График возрастает (рис.1).

    Если угол наклона прямой y = kx + b тупой, то угловой коэффициент является отрицательным числом. График убывает (рис.2).

    Если прямая параллельна оси абсцисс, то угол наклона прямой равен нулю. В этом случае угловой коэффициент прямой тоже равен нулю (так как тангенс нуля есть ноль). Уравнение прямой будет иметь вид y = b (рис.3).

    Если угол наклона прямой равен 90º (π/2), то есть она перпендикулярна оси абсцисс, то прямая задается равенством x = c , где c – некоторое действительное число (рис.4).

    Уравнение касательной к графику функции y = f (x ) в точке x о :


    Пример : Найдем уравнение касательной к графику функции f (x ) = x 3 – 2x 2 + 1 в точке с абсциссой 2.

    Решение .

    Следуем алгоритму.

    1) Точка касания x о равна 2. Вычислим f (x о ):

    f (x о ) = f (2) = 2 3 – 2 ∙ 2 2 + 1 = 8 – 8 + 1 = 1

    2) Находим f ′(x ). Для этого применяем формулы дифференцирования, изложенные в предыдущем разделе. Согласно этим формулам, х 2 = 2х , а х 3 = 3х 2 . Значит:

    f ′(x ) = 3х 2 – 2 ∙ 2х = 3х 2 – 4х .

    Теперь, используя полученное значение f ′(x ), вычислим f ′(x о ):

    f ′(x о ) = f ′(2) = 3 ∙ 2 2 – 4 ∙ 2 = 12 – 8 = 4.

    3) Итак, у нас есть все необходимые данные: x о = 2, f (x о ) = 1, f ′(x о ) = 4. Подставляем эти числа в уравнение касательной и находим окончательное решение:

    у = f (x о ) + f ′(x о ) (x – x о ) = 1 + 4 ∙ (х – 2) = 1 + 4х – 8 = –7 + 4х = 4х – 7.

    Ответ : у = 4х – 7.

    Инструкция

    Определяем угловой коэффициент касательной к кривой в точке М.
    Кривая, представляющая собой график функции y = f(x), непрерывна в некоторой окрестности точки М (включая саму точку М).

    Если значения f‘(x0) не существует, то либо касательной нет, либо она проходит вертикально. Ввиду этого, наличие производной функции в точке х0 обусловлено существованием невертикальной касательной, соприкасающейся с графиком функции в точке (х0, f(х0)). В этом случае угловой коэффициент касательной равен будет f»(х0). Таким образом, становится ясен геометрический смысл производной – расчет углового коэффициента касательной.

    Найдите значение абсциссы точки касания, которую обозначаются буквой «а». Если она совпадает с заданной точкой касательной, то «а» будет ее х-координате. Определите значение функции f(a), подставив в уравнение функции величину абсциссы.

    Определите первую производную уравнения функции f’(x) и подставьте в него значение точки «а».

    Возьмите общее уравнение касательной, которое определяется как y = f(a) = f (a)(x – a), и подставьте в него найденные значения a, f(a), f «(a). В результате будет найдено решение графика и касательной.

    Решите задачу иным способом, если заданная точка касательной не совпала с точкой касания. В этом случае необходимо в уравнение касательной вместо цифр подставить «а». После этого вместо букв «х» и «у» подставьте значение координат заданной точки. Решите получившееся уравнение, в котором «а» является неизвестной. Поставьте полученное значение в уравнение касательной.

    Составьте уравнение касательной с буквой «а», если в условии задачи задано уравнение функции и уравнение параллельной линии относительно искомой касательной. После этого необходимо производную функции , чтобы координату у точки «а». Подставьте соответствующее значение в уравнение касательной и решите функцию.

    В математике одним из параметров, описывающих положение прямой на декартовой плоскости координат, является угловой коэффициент этой прямой. Этот параметр характеризует наклон прямой к оси абцисс. Чтобы понять, как найти угловой коэффициент, сначала вспомним общий вид уравнения прямой в системе координат XY.

    В общем виде любую прямую можно представить выражением ax+by=c, где a, b и c — произвольные действительные числа, но обязательно a 2 + b 2 ≠ 0.

    Подобное уравнение с помощью несложных преобразований можно довести до вида y=kx+d, в котором k и d — действительные числа. Число k является угловым коэффициентом, а само уравнение прямой подобного вида называется уравнением с угловым коэффициентом. Получается, что для нахождения углового коэффициента, необходимо просто привести исходное уравнение к указанному выше виду. Для более полного понимания рассмотрим конкретный пример:

    Задача: Найти угловой коэффициент линии, заданной уравнением 36x — 18y = 108

    Решение: Преобразуем исходное уравнение.

    Ответ: Искомый угловой коэффициент данной прямой равен 2.

    В случае, если в ходе преобразований уравнения мы получили выражение типа x = const и не можем в результате представить y в виде функции x, то мы имеем дело с прямой, параллельной оси Х. Угловой коэффициент подобной прямой равен бесконечности.

    Для прямых, которых выражены уравнением типа y = const, угловой коэффициент равняется нулю. Это характерно для прямых, параллельных оси абцисс. Например:

    Задача: Найти угловой коэффициент линии, заданной уравнением 24x + 12y — 4(3y + 7) = 4

    Решение: Приведем исходное уравнение к общему виду

    24x + 12y — 12y + 28 = 4

    Из полученного выражения выразить y невозможно, следовательно угловой коэффициент данной прямой равен бесконечности, а сама прямая будет параллельна оси Y.

    Геометрический смысл

    Для лучшего понимания обратимся к картинке:

    На рисунке мы видим график функции типа y = kx. Для упрощения примем коэффициент с = 0. В треугольнике ОАВ отношение стороны ВА к АО будет равно угловому коэффициенту k. Вместе с тем отношение ВА/АО — это тангенс острого угла α в прямоугольном треугольнике ОАВ. Получается, что угловой коэффициент прямой равняется тангенсу угла, который составляет эта прямая с осью абцисс координатной сетки.

    Решая задачу, как найти угловой коэффициент прямой, мы находим тангенс угла между ней и осью Х сетки координат. Граничные случаи, когда рассматриваемая прямая параллельна осям координат, подтверждают вышенаписанное. Действительно для прямой, описанной уравнением y=const, угол между ней и осью абцисс равен нулю. Тангенс нулевого угла также равен нулю и угловой коэффициент тоже равен нулю.

    Для прямых, перпендикулярных оси абцисс и описываемых уравнением х=const, угол между ними и осью Х равен 90 градусов. Тангенс прямого угла равен бесконечности, так же и угловой коэффициент подобных прямых равен бесконечности, что подтверждает написанное выше.

    Угловой коэффициент касательной

    Распространенной, часто встречающейся на практике, задачей является также нахождение углового коэффициента касательной к графику функции в некоторой точке. Касательная — это прямая, следовательно к ней также применимо понятие углового коэффициента.

    Чтобы разобраться, как найти угловой коэффициент касательной, нам будет необходимо вспомнить понятие производной. Производная от любой функции в некоторой точке — это константа, численно равная тангенсу угла, который образуется между касательной в указанной точке к графику этой функции и осью абцисс. Получается, что для определения углового коэффициента касательной в точке x 0 , нам необходимо рассчитать значение производной исходной функции в этой точке k = f»(x 0). Рассмотрим на примере:

    Задача: Найти угловой коэффициент линии, касательной к функции y = 12x 2 + 2xe x при х = 0,1.

    Решение: Найдем производную от исходной функции в общем виде

    y»(0,1) = 24 . 0,1 + 2 . 0,1 . e 0,1 + 2 . e 0,1

    Ответ: Искомый угловой коэффициент в точке х = 0,1 равен 4,831

    В этой статье мы разберем все типы задач на нахождение

    Вспомним геометрический смысл производной : если к графику функции в точке проведена касательная, то коэффициент наклона касательной (равный тангенсу угла между касательной и положительным направлением оси ) равен производной функции в точке .


    Возьмем на касательной произвольную точку с координатами :


    И рассмотрим прямоугольный треугольник :


    В этом треугольнике

    Отсюда

    Это и есть уравнение касательной, проведенной к графику функции в точке .

    Чтобы написать уравнение касательной, нам достаточно знать уравнение функции и точку, в которой проведена касательная. Тогда мы сможем найти и .

    Есть три основных типа задач на составление уравнения касательной.

    1. Дана точка касания

    2. Дан коэффициент наклона касательной, то есть значение производной функции в точке .

    3. Даны координаты точки, через которую проведена касательная, но которая не является точкой касания.

    Рассмотрим каждый тип задач.

    1 . Написать уравнение касательной к графику функции в точке .

    .

    б) Найдем значение производной в точке . Сначала найдем производную функции

    Подставим найденные значения в уравнение касательной:

    Раскроем скобки в правой части уравнения. Получим:

    Ответ: .

    2 . Найти абсциссы точек, в которых касательные к графику функции параллельны оси абсцисс.

    Если касательная параллельна оси абсцисс, следовательно угол между касательной и положительным направлением оси равен нулю, следовательно тангенс угла наклона касательной равен нулю. Значит, значение производной функции в точках касания равно нулю.

    а) Найдем производную функции .

    б) Приравняем производную к нулю и найдем значения , в которых касательная параллельна оси :

    Приравняем каждый множитель к нулю, получим:

    Ответ: 0;3;5

    3 . Написать уравнения касательных к графику функции , параллельных прямой .

    Касательная параллельна прямой . Коэффициент наклона этой прямой равен -1. Так как касательная параллельна этой прямой, следовательно, коэффициент наклона касательной тоже равен -1. То есть мы знаем коэффициент наклона касательной , а, тем самым, значение производной в точке касания .

    Это второй тип задач на нахождение уравнения касательной.

    Итак, у нас дана функция и значение производной в точке касания.

    а) Найдем точки, в которых производная функции равна -1.

    Сначала найдем уравнение производной.

    Приравняем производную к числу -1.

    Найдем значение функции в точке .

    (по условию)

    .

    б) Найдем уравнение касательной к графику функции в точке .

    Найдем значение функции в точке .

    (по условию).

    Подставим эти значения в уравнение касательной:

    .

    Ответ:

    4 . 2}»>. Мы получили под корнем отрицательное число, равенство не верно, и точка не принадлежит графику функции и не является точкой касания.

    Это последний тип задач на нахождение уравнения касательной. Первым делом нам нужно найти абсциссу точки касания .

    Найдем значение .

    Пусть — точка касания. Точка принадлежит касательной к графику функции . Если мы подставим координаты этой точки в уравнение касательной, то получим верное равенство:

    .

    Значение функции в точке равно .

    Найдем значение производной функции в точке .

    Сначала найдем производную функции . Это .

    Производная в точке равна .

    Подставим выражения для и в уравнение касательной. Получим уравнение относительно :

    Решим это уравнение.

    Сократим числитель и знаменатель дроби на 2:

    Приведем правую часть уравнения к общему знаменателю. Получим:

    Упростим числитель дроби и умножим обе части на — это выражение строго больше нуля. 2} {8-3x_0>=0} }}{ }»>

    Решим первое уравнение.

    Решим квадратное уравнение, получим

    Второй корень не удовлетворяет условию title=»8-3x_0>=0″>, следовательно, у нас только одна точка касания и её абсцисса равна .

    Напишем уравнение касательной к кривой в точке . Для этого подставим значение в уравнение — мы его уже записывали.

    Ответ:
    .

    Пусть дана функция f , которая в некоторой точке x 0 имеет конечную производную f (x 0). Тогда прямая, проходящая через точку (x 0 ; f (x 0)), имеющая угловой коэффициент f ’(x 0), называется касательной.

    А что будет, если производная в точке x 0 не существует? Возможны два варианта:

    1. Касательная к графику тоже не существует. Классический пример — функция y = |x | в точке (0; 0).
    2. Касательная становится вертикальной. Это верно, к примеру, для функции y = arcsin x в точке (1; π /2).

    Уравнение касательной

    Всякая невертикальная прямая задается уравнением вида y = kx + b , где k — угловой коэффициент. Касательная — не исключение, и чтобы составить ее уравнение в некоторой точке x 0 , достаточно знать значение функции и производной в этой точке.

    Итак, пусть дана функция y = f (x ), которая имеет производную y = f ’(x ) на отрезке . Тогда в любой точке x 0 ∈ (a ; b ) к графику этой функции можно провести касательную, которая задается уравнением:

    y = f ’(x 0) · (x − x 0) + f (x 0)

    Здесь f ’(x 0) — значение производной в точке x 0 , а f (x 0) — значение самой функции.

    Задача. Дана функция y = x 3 . Составить уравнение касательной к графику этой функции в точке x 0 = 2.

    Уравнение касательной: y = f ’(x 0) · (x − x 0) + f (x 0). Точка x 0 = 2 нам дана, а вот значения f (x 0) и f ’(x 0) придется вычислять.

    Для начала найдем значение функции. Тут все легко: f (x 0) = f (2) = 2 3 = 8;
    Теперь найдем производную: f ’(x ) = (x 3)’ = 3x 2 ;
    Подставляем в производную x 0 = 2: f ’(x 0) = f ’(2) = 3 · 2 2 = 12;
    Итого получаем: y = 12 · (x − 2) + 8 = 12x − 24 + 8 = 12x − 16.
    Это и есть уравнение касательной.

    Задача. Составить уравнение касательной к графику функции f (x ) = 2sin x + 5 в точке x 0 = π /2.

    В этот раз не будем подробно расписывать каждое действие — укажем лишь ключевые шаги. Имеем:

    f (x 0) = f (π /2) = 2sin (π /2) + 5 = 2 + 5 = 7;
    f ’(x ) = (2sin x + 5)’ = 2cos x ;
    f ’(x 0) = f ’(π /2) = 2cos (π /2) = 0;

    Уравнение касательной:

    y = 0 · (x − π /2) + 7 ⇒ y = 7

    В последнем случае прямая оказалась горизонтальной, т.к. ее угловой коэффициент k = 0. Ничего страшного в этом нет — просто мы наткнулись на точку экстремума.

    «Нахождение углового коэффициэнта касательной к графику функции в указанной точке. Составление уравнения касательной.»

    Цель: Составить уравнение касательной к графику функции в заданной точке.

    Теоретический материал:

    Углом наклона прямой y = kx+b называют угол , отсчитываемый от положительного направления оси абсцисс до прямой y = kx+b в положительном направлении (то есть, против часовой стрелки). Угловым коэффициентом прямой y = kx+b называют числовой коэффициент k. Угловой коэффициент прямой равен тангенсу угла наклона прямой, то есть, .

    Угол наклона прямой равен нулю, когда прямая параллельна оси абсцисс. В этом случае нулю равен и угловой коэффициент, так как тангенс нуля есть ноль. Следовательно, уравнение прямой будет иметь вид y = b.

    Когда угол наклона прямой y = kx+b является острым ( ), то угловой коэффициент k является положительным числом (так как тангенс острого угла принимает положительные значения ) и указывает на возрастание графика прямой.

    В случае, когда прямая располагается перпендикулярно оси абсцисс (параллельно оси ординат) и задается равенством x = c, где c — некоторое действительное число.

    Когда угол наклона прямой y = kx+b является тупым ( ), то угловой коэффициент k является отрицательным числом и указывает на убывание графика прямой.

    Касательной к графику функции y = f(x) в точке называют прямую, проходящую через точку , с отрезком которой практически сливается график функции при значениях х сколь угодно близких к . Для этого покажем, что будет происходить с секущей АВ, если точку В бесконечно приближать к точке А.

    Рисунок ниже иллюстрирует этот процесс.

    Секущая АВ (показана синей пунктирной прямой) будет стремиться занять положение касательной прямой (показана синей сплошной линией), угол наклона секущей (показан красной прерывистой дугой) будет стремиться к углу наклона касательной (изображен красной сплошной дугой). Таким образом, касательная к графику функции y = f(x) в точке А – это предельное положение секущей AB при .

    Геометрический смысл производной функции в точке.

    Рассмотрим секущую АВ графика функции y = f(x) такую, что точки А и В имеют соответственно координаты и , где — приращение аргумента. Обозначим через приращение функции. Отметим все на чертеже:

    Из прямоугольного треугольника АВС имеем . Так как по определению касательная – это предельное положение секущей, то . Вспомним определение производной функции в точке: производной функции y = f(x) в точке называется предел отношения приращения функции к приращению аргумента при , обозначается . Следовательно, , где — угловой коэффициент касательной. Таким образом, существование производной функции y = f(x) в точке эквивалентно существованию касательной к графику функции y = f(x) в точке касания , причем угловой коэффициент касательной равен значению производной в точке , то есть

    .

    Составление уравнения касательной прямой

    Для записи уравнения любой прямой на плоскости достаточно знать ее угловой коэффициент и точку, через которую она проходит. Уравнение касательной к графику функции y = f(x) в точке имеет вид:

    .

    Алгоритм составления уравнения касательной к графику функции y = f(x)

    1. Обозначить буквой a абсциссу точки касания. 2. Найти f(a). 3. Найти f ‘(x) и f ‘(a). 4. Подставить найденные числа a, f(a), f ‘(a) в общее уравнение касательной y — f(a) = f ‘(a)(x – a).

    Примеры составления уравнения касательной.

    Пример 1. Составьте уравнение касательной в точке M(3; – 2) к графику функции .

    Решение. Точка M(3; – 2) является точкой касания, так как

    1. a = 3 – абсцисса точки касания. 2. f(3) = – 2. 3. f ‘(x) = x2 – 4, f ‘(3) = 5. y = – 2 + 5(x – 3), y = 5x – 17 – уравнение касательной.

    Задание для практической работы по теме «Нахождение углового коэффициента касательной к графику функции в указанной точке. Составление уравнения касательной». Составить уравнения касательных к графикам функции в заданной точке с абсциссой а=2:

    Вариант 1

    Вариант 2

    Вариант 3

    Вариант 4

    Уровень А. Уровень А.

    Y=3x3-x

    Уровень B Уровень B

    Уровень А. Уровень А.

    Y=-x3+x

    Уровень B Уровень B

    Уровень А. Уровень А.

    Y=2x2-8x

    Уровень B Уровень B

    Уровень А. Уровень А.

    Y=-3x2+12x

    Уровень B Уровень B

    Вариант 5

    Вариант 6

    Вариант 7

    Вариант 8

    Уровень А. Уровень А.

    Y=x2+5x+4

    Уровень B Уровень B

    Уровень А. Уровень А.

    Y=-x2+2x+15

    Уровень B Уровень B

    Уровень А. Уровень А.

    Y=1/3x3-9

    Уровень B Уровень B

    Уровень А. Уровень А.

    Y=x3-3x

    Уровень B Уровень B

    Практическое Занятие №3

    Найти наклон линии, касательной к кривой в заданной точке

    Найти наклон линии, касательной к кривой в заданной точке — Предварительное вычисление

    —>

    • Войти
    • Биографии репетитора
    • Подготовка к тесту
      СРЕДНЯЯ ШКОЛА
      • ACT Репетиторство
      • SAT Репетиторство
      • Репетиторство PSAT
      • ASPIRE Репетиторство
      • ШСАТ Репетиторство
      • Репетиторство STAAR
      ВЫСШАЯ ШКОЛА
      • Репетиторство MCAT
      • Репетиторство GRE
      • Репетиторство по LSAT
      • Репетиторство по GMAT
      К-8
      • Репетиторство AIMS
      • Репетиторство по HSPT
      • Репетиторство ISEE
      • Репетиторство ISAT
      • Репетиторство по SSAT
      • Репетиторство STAAR
      Поиск 50+ тестов
    • Академическое обучение
      репетиторство по математике
      • Алгебра
      • Исчисление
      • Элементарная математика
      • Геометрия
      • Предварительный расчет
      • Статистика
      • Тригонометрия
      Репетиторство по естественным наукам
      • Анатомия
      • Биология
      • Химия
      • Физика
      • Физиология
      иностранные языки
      • французский
      • немецкий
      • Латинский
      • Китайский мандарин
      • Испанский
      начальное обучение
      • Чтение
      • Акустика
      • Элементарная математика
      прочие
      • Бухгалтерский учет
      • Информатика
      • Экономика
      • Английский
      • Финансы
      • История
      • Письмо
      • Лето
      Поиск по 350+ темам
    • О
      • Обзор видео
      • Процесс выбора наставника
      • Онлайн-репетиторство
      • Мобильное обучение
      • Мгновенное обучение
      • Как мы работаем
      • Наша гарантия
      • Влияние репетиторства
      • Обзоры и отзывы
      • Освещение в СМИ
      • О преподавателях университета

    Мы открыты в субботу и воскресенье!

    Звоните прямо сейчас, чтобы записаться на обучение:

    (888) 888-0446

    Все ресурсы Precalculus

    12 диагностических тестов 380 практических тестов Вопрос дня Карточки Учитесь по концепции

    Справка по предварительному исчислению » Вводный расчет » Касательные к кривой » Найти наклон прямой, касательной к кривой в заданной точке

    Найти наклон прямой в данной точке.

    Возможные ответы:

    Правильный ответ:

    Пояснение:

    Сначала найдите наклон касательной к прямой, взяв производную.

    Используя экспоненциальное правило, мы получаем следующее,

     

    .

    Затем подставьте 1 в уравнение, поскольку 1 — это точка, в которой нужно найти наклон.

    .

    Сообщить об ошибке

    Найдите наклон следующего выражения в точке 

    .

    Возможные ответы:

    Правильный ответ:

    Объяснение:

    Один из способов найти уклон в заданной точке — найти производную. В этом случае мы можем взять производную от y по x и подставить желаемое значение для x.

    Используя экспоненциальное правило, мы получаем следующую производную,

    .

    Подстановка x=2 из точки 2,3 дает нам окончательный уклон,

    Таким образом, наш уклон в конкретной точке равен .

    Обратите внимание, что в этом случае использование координаты y не требуется.

    Сообщить об ошибке

    Найти наклон касательной функции при заданном значении

    при

    .

    Возможные ответы:

    Правильный ответ:

    Объяснение:

    Чтобы найти наклон касательной функции при заданном значении, оцените первую производную для заданного значения.

    Первая производная составляет

    и для этой функции

    и

    , поэтому наклон составляет

    Отчет о ошибке

    Найдите наклон линии касательной функции в данном значении.

    на

    .

     

     

    Возможные ответы:

    Правильный ответ:

    Объяснение:

    Чтобы найти наклон касательной функции при заданном значении, вычислите первую производную для заданного значения.

    Первая производная

    и для этой функции

    и подставляя конкретное значение x мы получаем,

    Итак, наклон равен

    .

    Сообщить об ошибке

    Рассмотрим функцию . Каков наклон линии, касательной к графику в точке ?

    Возможные ответы:

    Правильный ответ:

    Объяснение:

    Вычислить производную от  с помощью правил производной. Производная функция определяет наклон в любой точке исходной функции.

    Производная: 

    С заданной точкой , . Подставьте это значение в производную функцию, чтобы определить наклон в этой точке.

    Наклон касательной, пересекающей точку, равен .

    Сообщить об ошибке

    Уведомление об авторских правах

    Посмотреть репетиторов по предварительному исчислению

    Элисон
    Сертифицированный репетитор

    Университет Сент-Томас, бакалавр гуманитарных наук, биологии, общего профиля. Университет Миннесоты, бакалавр наук, сестринское дело (RN).

    Просмотр репетиторов по математике

    Harrison
    Сертифицированный преподаватель

    Колледж Гамильтона, бакалавр искусств, математика. Университет Коннектикута, магистр прикладной математики.

    View Pre-Calculus Tutors

    Magdy
    Сертифицированный преподаватель

    Каирский университет, Египет, бакалавр электротехники. Государственный университет Нью-Мексико, главный кампус, доктор фил…

    Все ресурсы Precalculus

    12 диагностических тестов 380 практических тестов Вопрос дня Карточки Учитесь по концепции

    Исчисление I. Касательные линии и скорости изменения

    Показать мобильное уведомление Показать все примечания Скрыть все примечания

    Мобильное уведомление

    Похоже, вы используете устройство с «узкой» шириной экрана ( т. е. вы, вероятно, используете мобильный телефон). Из-за характера математики на этом сайте лучше всего просматривать в ландшафтном режиме. Если ваше устройство не находится в ландшафтном режиме, многие уравнения будут отображаться сбоку вашего устройства (должна быть возможность прокрутки, чтобы увидеть их), а некоторые пункты меню будут обрезаны из-за узкой ширины экрана.

    Раздел 2-1: Касательные линии и скорости изменения

    В этом разделе мы рассмотрим две довольно важные проблемы в изучении исчисления. Есть две причины обратить внимание на эти проблемы сейчас.

    Во-первых, обе эти проблемы приведут нас к изучению пределов, что, в конце концов, является темой этой главы. Рассмотрение этих проблем здесь позволит нам начать понимать, что такое предел и что он может рассказать нам о функции.

    Во-вторых, проблема скорости изменения, которую мы собираемся рассмотреть, является одним из наиболее важных понятий, с которыми мы столкнемся во второй главе этого курса. На самом деле, это, наверное, одна из самых важных концепций, с которыми мы столкнемся на протяжении всего курса. Итак, глядя на это сейчас, мы начинаем думать об этом с самого начала.

    Касательные линии

    Первая задача, которую мы собираемся рассмотреть, — это задача касательной линии. Прежде чем приступить к этой проблеме, вероятно, было бы лучше определить касательную линию.

    Касательной к функции \(f(x)\) в точке \(x = a\) называется прямая, которая касается графика функции в рассматриваемой точке и является «параллельной» (в некоторых путь) к графику в этой точке. Взгляните на график ниже.

    На этом графике линия является касательной в указанной точке, потому что она только касается графика в этой точке, а также «параллельна» графику в этой точке. Точно так же во второй показанной точке линия просто касается графика в этой точке, но она не «параллельна» графику в этой точке и, следовательно, не является касательной к графику в этой точке.

    Во второй показанной точке (точке, где линия не является касательной) мы иногда будем называть линию секущей линией .

    Мы уже пару раз использовали слово «параллельный», и нам, вероятно, следует быть с ним немного осторожнее. В общем, мы будем думать о линии и графике как о параллельных в точке, если они оба движутся в одном направлении в этой точке. Итак, в первой точке выше график и линия движутся в одном направлении, поэтому мы будем говорить, что они параллельны в этой точке. С другой стороны, во второй точке линия и график не движутся в одном направлении, поэтому в этой точке они не параллельны. 92}\) в \(х=1\).

    Показать решение

    Из алгебры мы знаем, что для нахождения уравнения прямой нужны либо две точки на прямой, либо одна точка на прямой и наклон прямой. Поскольку мы знаем, что находимся после касательной, у нас есть точка, которая находится на этой линии. Касательная и график функции должны соприкасаться в точке \(x\) = 1, поэтому точка \(\left( {1,f\left( 1 \right)} \right) = \left( {1,13 } \right)\) должен быть в строке.

    Теперь мы подошли к проблеме. Это все, что мы знаем о касательной. Чтобы найти касательную, нам нужна либо вторая точка, либо наклон касательной. Поскольку единственная причина, по которой нам нужна вторая точка, состоит в том, чтобы позволить нам найти наклон касательной, давайте просто сконцентрируемся на том, сможем ли мы определить наклон касательной.

    На данный момент все, что мы можем сделать, это получить оценку наклона касательной, но если мы сделаем это правильно, мы сможем получить оценку, которая на самом деле является фактический наклон касательной. Мы сделаем это, начав с точки, которая нам нужна, назовем ее \(P = \left({1,13} \right)\). Затем мы выберем другую точку, лежащую на графике функции, назовем ее \(Q = \left( {x,f\left( x \right)} \right)\).

    Для аргументации возьмем \(x = 2\) и тогда вторая точка будет \(Q = \left( {2,7} \right)\). Ниже приведен график функции, касательная и секущая, соединяющая \(P\) и \(Q\).

    Из этого графика видно, что секущая и касательная в чем-то похожи, поэтому наклон секущей должен быть несколько близок к фактическому наклону касательной. Итак, в качестве оценки наклона касательной мы можем использовать наклон секущей, назовем его \({m_{PQ}}\), то есть

    \[{m_{PQ}} = \frac{{f\left( 2 \right) — f\left( 1 \right)}}{{2 — 1}} = \frac{{7 — 13}}{ 1} = — 6\]

    Теперь, если бы нас не слишком интересовала точность, мы могли бы сказать, что этого достаточно, и использовать это как оценку наклона касательной. Однако хотелось бы, чтобы оценка была хоть как-то близка к фактическому значению. Таким образом, чтобы получить лучшую оценку, мы можем взять \(x\), который ближе к \(x = 1\), и повторить вышеописанную работу, чтобы получить новую оценку наклона. Затем мы могли бы взять третье значение \(x\) еще ближе и получить еще лучшую оценку.

    Другими словами, по мере того, как мы приближаем \(Q\) к \(P\), наклон секущей, соединяющей \(Q\) и \(P\), должен становиться все ближе и ближе к наклону касательной линии. Если вы просматриваете это в Интернете, на изображении ниже показан этот процесс.

    Как вы можете видеть (к сожалению, анимация не будет работать на всех программах просмотра PDF), по мере того, как мы перемещали \(Q\) все ближе и ближе к \(P\), секущие линии начинают все больше и больше походить на касательную и поэтому приблизительные наклоны ( т.е. наклоны секущих) все ближе и ближе к точному наклону. Кроме того, не беспокойтесь о том, как я получил точные или приблизительные наклоны. Вскоре мы вычислим приблизительные наклоны и сможем вычислить точный наклон на нескольких участках.

    На этом рисунке мы рассмотрели только \(Q\), которые были справа от \(P\), но мы могли бы так же легко использовать \(Q\), которые были слева от \ (P\), и мы получили бы такие же результаты. На самом деле, мы всегда должны смотреть на \(Q\), которые находятся по обе стороны от \(P\). В этом случае одно и то же происходит по обе стороны от \(P\). Однако в конечном итоге мы увидим, что этого не должно произойти. Следовательно, мы всегда должны смотреть на то, что происходит по обе стороны от рассматриваемой точки при выполнении такого рода процесса. 2}}}{{х — 1}}\]

    Теперь давайте выберем несколько значений \(x\), все ближе и ближе к \(x = 1\), подключим и получим наклоны.

    \(х\) \({м_{PQ}}\) \(х\) \({м_{PQ}}\)
    2 -6 0 -2
    1,5 -5 0,5 -3
    1.1 -4,2 0,9 -3,8
    1,01 -4.02 0,99 -3,98
    1,001 -4.002 0,999 -3,998
    1. 0001 -4.0002 0,9999 -3,9998

    Итак, если мы возьмем \(x\) справа от 1 и переместим их очень близко к 1, окажется, что наклон секущих приближается к -4. Точно так же, если мы возьмем \(x\) слева от 1 и переместим их очень близко к 1, наклон секущих снова окажется приближающимся к -4.

    Основываясь на этом свидетельстве, кажется, что наклоны секущих приближаются к -4, когда мы приближаемся к \(x = 1\), поэтому мы оценим, что наклон касательной также равен -4. Как отмечалось выше, это правильное значение, и в конечном итоге мы сможем это доказать.

    Теперь уравнение прямой, проходящей через \[\left( {a,f\left( a \right)} \right)\] задается как

    \[y = f\left( a \right) + m\left( {x — a} \right)\] 92}\) в \(x=1\) равно

    \[y = 13 — 4\влево( {x — 1} \вправо) = — 4x + 17\]

    Следует отметить несколько важных моментов в нашей работе. Во-первых, мы рассмотрели точки, находящиеся по обе стороны от \(x = 1\). В такого рода процессах важно никогда не предполагать, что то, что происходит на одной стороне точки, будет происходить и на другой стороне. Мы всегда должны смотреть на то, что происходит по обе стороны от точки. В этом примере мы могли бы набросать график и, исходя из этого, предположить, что то, что происходит с одной стороны, будет происходить и с другой, но обычно у нас не будет графиков перед собой или мы не сможем легко их получить.

    Далее, обратите внимание, что когда мы говорим, что собираемся приблизиться к рассматриваемой точке, мы имеем в виду, что собираемся приблизиться очень близко, и мы также использовали больше, чем пару точек. Мы никогда не должны пытаться определить тренд, основываясь на паре точек, которые на самом деле не так уж близки к рассматриваемой точке.

    Следующее, на что следует обратить внимание, — это скорее предупреждение, чем что-либо еще. Значения \({m_{PQ}}\) в этом примере были довольно «хорошими», и было довольно ясно, к какому значению они приближаются после пары вычислений. В большинстве случаев этого не будет. Большинство значений будут гораздо «беспорядочнее», и вам часто потребуется довольно много вычислений, чтобы получить оценку. Вы всегда должны использовать не менее четырех точек с каждой стороны, чтобы получить оценку. Двух баллов никогда не бывает достаточно для получения хорошей оценки, и трех баллов также часто недостаточно для получения хорошей оценки. Как правило, вы выбираете точки все ближе и ближе к точке, на которую смотрите, пока изменение значения между двумя последовательными точками не станет очень маленьким.

    Наконец, нам нужно было выяснить, что происходило при \(x = 1\), и мы не могли подставить \(x = 1\) в нашу формулу для наклона. Несмотря на это ограничение, мы смогли получить некоторую информацию о том, что происходит в точке \(x = 1\), просто взглянув на то, что происходит вокруг точки \(x = 1\). Это важнее, чем вы можете себе представить на первый взгляд, и мы подробно обсудим этот момент в следующих разделах.

    Прежде чем двигаться дальше, давайте кратко рассмотрим, что мы сделали в приведенном выше примере. Мы хотели, чтобы касательная шла к \(f\left( x \right)\) в точке \(x = a\). Во-первых, мы знаем, что точка \(P = \left( {a,f\left( a \right)} \right)\) будет на касательной. Затем мы возьмем вторую точку на графике функции, назовем ее \(Q = \left( {x,f\left( x \right)} \right)\) и вычислим наклон кривой линия, соединяющая \(P\) и \(Q\) следующим образом,

    \[{m_{PQ}} = \frac{{f\left( x \right) — f\left( a \right)}}{{x — a}}\]

    Затем мы берем значения \(x\), которые все ближе и ближе к \(x = a\) (убедившись, что мы смотрим на \(x\) по обе стороны от \(x = a\) и используйте этот список значений для оценки наклона касательной, \(m\)

    Тогда касательная будет равна

    \[y = f\left( a \right) + m\left( {x — a} \right)\]

    Скорость изменения

    Следующая проблема, которую нам нужно рассмотреть, это проблема скорости изменения. Как упоминалось ранее, это окажется одной из самых важных концепций, которые мы будем рассматривать на протяжении всего курса.

    Здесь мы собираемся рассмотреть функцию \(f\left( x \right)\), представляющую некоторую величину, которая изменяется при изменении \(x\). Например, возможно, \(f\left( x \right)\) представляет собой количество воды в накопительном баке через \(x\) минут. Или, может быть, \(f\left( x \right)\) — это расстояние, пройденное автомобилем за \(x\) часов. В обоих этих примерах мы использовали \(x\) для представления времени. Конечно, \(x\) не обязательно должно представлять время, но это дает примеры, которые легко визуализировать.

    Здесь мы хотим определить, насколько быстро \(f\left( x \right)\) меняется в какой-то момент, скажем, \(x = a\). Это называется мгновенной скоростью изменения или иногда просто скоростью изменения \(f\left( x \right)\) в \(x = a\).

    Как и в задаче о касательной линии, все, что мы сейчас можем сделать, — это оценить скорость изменения. Итак, давайте продолжим с приведенными выше примерами и будем думать о \(f\left( x \right)\) как о чем-то, что изменяется во времени, а \(x\) является измерением времени. Опять же, \(x\) не обязательно представляет время, но это немного облегчит объяснение. Хотя мы не можем вычислить мгновенную скорость изменения, мы можем найти среднюю скорость изменения.

    Чтобы вычислить среднюю скорость изменения \(f\left( x \right)\) в \(x = a\), все, что нам нужно сделать, это выбрать другую точку, скажем, \(x\), а затем средняя скорость изменения будет

    \[\begin{align*}А.Р.К. & = \frac{{{\mbox{изменение}}f\left( x \right)}}{{{\mbox{изменение}}x}}\\ & = \frac{{f\left( x \right) — f\left( a \right)}}{{x — a}}\end{align*}\]

    Затем, чтобы оценить мгновенную скорость изменения при \(x = a\), все, что нам нужно сделать, это выбрать значения \(x\), все ближе и ближе к \(x = a\) (не забывайте чтобы выбрать их по обе стороны от \(x = a\)) и вычислить значения \(A.R.C.\). Затем мы можем оценить мгновенную скорость изменения на основе этого. 92} + 25}}{{т — 5}}\]

    Чтобы оценить мгновенную скорость изменения объема при \(t = 5\), нам просто нужно выбрать значения \(t\), которые все ближе и ближе к \(t = 5\). Вот таблица значений \(t\) и средней скорости изменения этих значений.

    \(т\) \(АРК\) \(т\) \(АРК\)
    6 25,0 4 7,0
    5,5 19,75 4,5 10,75
    5.1 15,91 4,9 14.11
    5.01 15.0901 4,99 14.9101
    5.001 15.009001 4,999 14.9
    5.0001 15. 000 4,9999 14.991

    Итак, из этой таблицы видно, что средняя скорость изменения приближается к 15, и поэтому мы можем оценить, что мгновенная скорость изменения на данный момент равна 15.

    Итак, что же это говорит нам об объеме в точке \(t = 5\)? Положим несколько единиц на ответ сверху. Это может помочь нам увидеть, что происходит с томом в этот момент. Предположим, что единицы объема были в см 3 . Тогда единицы скорости изменения (как средней, так и мгновенной) равны см 3 /час.

    Мы подсчитали, что при \(t = 5\) объем изменяется со скоростью 15 см 3 /час. Это означает, что при \(t = 5\) объем изменяется так, что, если бы скорость была постоянной, то через час в баллоне было бы на 15 см 3 воздуха больше, чем было при \( т = 5\).

    Однако здесь нужно быть осторожным. На самом деле через час в воздушном шаре, вероятно, не останется еще 15 см 3 воздуха. Скорость, с которой изменяется объем, как правило, непостоянна, поэтому мы не можем реально определить, каким будет объем через следующий час. Что мы можем сказать, так это то, что объем увеличивается, поскольку мгновенная скорость изменения положительна, и если бы у нас были скорости изменения для других значений \(t\), мы могли бы сравнить числа и посмотреть, быстрее ли скорость изменения или медленнее в других точках.

    Например, при \(t = 4\) мгновенная скорость изменения равна 0 см 3 /час, а при \(t = 3\) мгновенная скорость изменения равна -9 см 3 /час. Мы предоставим вам проверить эти скорости изменения. На самом деле, это было бы хорошим упражнением, чтобы увидеть, сможете ли вы построить таблицу значений, которая подтвердит наши утверждения об этих темпах изменений.

    Итак, вернемся к примеру. При \(t = 4\) скорость изменения равна нулю, поэтому в этот момент времени объем вообще не меняется. Это не значит, что он не изменится в будущем. Это просто означает, что ровно при \(t = 4\) громкость не меняется. Точно так же в точке \(t = 3\) объем уменьшается, поскольку скорость изменения в этой точке отрицательна. Мы также можем сказать, что, независимо от аспектов увеличения/уменьшения скорости изменения, объем воздушного шара изменяется быстрее при \(t = 5\), чем при \(t = 3\), поскольку 15 больше чем 9.

    Мы еще поговорим о скоростях изменений, когда перейдем к следующей главе.

    Задача о скорости

    Давайте кратко рассмотрим задачу о скорости. Многие книги по математическому анализу рассматривают это как отдельную проблему. Однако нам нравится думать об этом как о частном случае проблемы скорости изменения. В задаче о скорости нам дана функция положения объекта \(f\left( t \right)\), которая дает положение объекта в момент времени \(t\). Затем, чтобы вычислить мгновенную скорость объекта, нам просто нужно вспомнить, что скорость — это не что иное, как скорость изменения положения.

    Другими словами, чтобы оценить мгновенную скорость, мы должны сначала вычислить среднюю скорость,

    \[\begin{align*}А. В. & = \frac{{{\mbox{изменение положения}}}}{{{\mbox{пройденное время}}}}\\ & \\ & = \frac{{f\left( t \right) — f \left( a \right)}}{{t — a}}\end{align*}\]

    , а затем подбирайте значения \(t\) все ближе и ближе к \(t = a\) и используйте эти значения для оценки мгновенной скорости.

    Изменение обозначения

    Последнее, что нам нужно сделать в этом разделе, прежде чем двигаться дальше. Основная цель этого раздела состояла в том, чтобы познакомить нас с парой ключевых концепций и идей, которые мы увидим в первой части этого курса, а также помочь нам начать путь к ограничениям.

    Прежде чем мы официально перейдем к ограничениям, давайте вернемся назад и проделаем небольшую работу, которая свяжет обе (или все три, если включить скорость как отдельную проблему) проблемы в более общую концепцию.

    Во-первых, обратите внимание, что независимо от того, нужна ли нам касательная, мгновенная скорость изменения или мгновенная скорость, каждый из них сводился к использованию одной и той же формулы. А именно,

    \begin{equation}\frac{{f\left( x \right) — f\left( a \right)}}{{x — a}} \label{eq:eq1}\end{equation}

    Это должно означать, что все три проблемы на самом деле являются одной и той же проблемой. На самом деле это так, как мы увидим в следующей главе. Мы действительно работаем над одной и той же проблемой в каждом из этих случаев, единственная разница заключается в интерпретации результатов.

    При подготовке к следующему разделу, где мы обсудим это более подробно, нам нужно быстро изменить обозначения. Это проще сделать здесь, так как мы уже потратили изрядное количество времени на эти проблемы.

    Во всех этих задачах мы хотели определить, что происходит в точке \(x = a\). Для этого мы выбрали другое значение \(x\) и подключили его к \(\eqref{eq:eq1}\). Для того, что мы делали здесь, это, вероятно, самый интуитивный способ сделать это. Однако, когда мы начнем рассматривать эти проблемы как одну проблему, \(\eqref{eq:eq1}\) не будет лучшей формулой для работы.

    Вместо этого мы сначала определим, как далеко от \(x = a\) мы хотим двигаться, а затем определим нашу новую точку на основе этого решения. Итак, если мы хотим переместиться на расстояние \(h\) от \(x = a\), новая точка будет \(x = a + h\). Это показано на эскизе ниже.

    Как мы видели в нашей работе выше, важно брать значения \(x\), которые являются обеими сторонами \(x = a\). Этот способ выбора нового значения \(x\) сделает это за нас, как мы можем видеть на скетче выше. Если \(h > 0\), мы получим значение \(x\), которое находится справа от \(x = a\), а если \(h < 0\), мы получим значения \(x\) которые находятся слева от \(x = a\), и оба задаются как \(x = a + h\).

    Теперь, с этим новым способом получения второго значения \(x\) \(\eqref{eq:eq1}\) станет

    \[\frac{{f\left( x \right) — f\left( a \right)}}{{x — a}} = \frac{{f\left( {a + h} \right) — f\left( a \right)}}{{a + h — a}} = \frac{{f\left( {a + h} \right) — f\left( a \right)}}{h} \]

    Теперь это для конкретного значения \(x\), т. е. \(x = a\), и мы редко будем рассматривать их при конкретных значениях \(x\). Итак, мы делаем последний шаг в приведенном выше уравнении и заменяем \(a\) на \(x\), чтобы получить

    \[\frac{{f\left( {x + h} \right) — f\left( x \right)}}{h}\]

    Это дает нам формулу для общего значения \(x\), и на первый взгляд может показаться, что это будет слишком сложный способ работы с этим материалом. Однако, как мы увидим, часто будет проще иметь дело с этой формой, чем с исходной формой, \(\eqref{eq:eq1}\).

    Касательная линия — уравнение, наклон, горизонталь

    «Касательная линия» — одно из наиболее важных применений дифференцирования. Слово «тангенс» происходит от латинского слова «tangere», что означает «касаться». Касательная линия касается кривой в точке на кривой. Таким образом, чтобы найти уравнение касательной линии, нам нужно знать уравнение кривой (которое задается функцией) и точку, в которой проведена касательная.

    Давайте посмотрим, как найти наклон и уравнение касательной вместе с несколькими решенными примерами. Кроме того, давайте посмотрим, как найти уравнение касательной к параметрической кривой и полярной кривой.

    1. Что такое касательная?
    2. Наклон касательной
    3. Уравнение касательной линии
    4. Аппроксимация касательной линии
    5. Уравнение касательной к параметрической кривой
    6. Касательная линия полярной кривой
    7. Часто задаваемые вопросы о касательной линии

    Что такое касательная?

    Касательная линия кривой в данной точке — это линия, которая только касается кривой (функции) в этой точке. Касательная линия в исчислении может касаться кривой в любой другой точке (точках), а также может пересекать график в какой-либо другой точке (точках). Точка, в которой проводится касательная, известна как «точка касания». Мы можем видеть касательную окружности, нарисованную здесь.

    Если прямая проходит через две точки кривой, но не касается кривой ни в одной из точек, то она НЕ является касательной к кривой в каждой из двух точек. В этом случае линия называется секущей. Здесь мы можем увидеть несколько примеров касательных и секущих линий. Ниже показана секущая линия PQ, которая НЕ является касательной ни в P, ни в Q.

    Примеры касательной

    Вот типичный пример касательной, которая касается кривой точно в одной точке.

    Как мы узнали ранее, касательная может касаться кривой в нескольких точках. Вот пример.

    Опять же, касательная к кривой, проведенной в точке, может пересечь кривую и в какой-то другой точке. Вот касательная, проведенная в точке P, но пересекающая кривую в какой-то другой точке Q.

    Приведенную выше прямую PQ можно также назвать секущей. Секущая линия также может проходить через любые две точки кривой без необходимости касаться кривой в каждой из двух точек.

    Наклон касательной линии

    Рассмотрим кривую, представленную функцией f(x). Также рассмотрим секущую, проходящую через две точки кривой P (x 0 , f(x 0 )) и Q (x 0 + h, f(x 0 + h)) . т. е. P и Q находятся на расстоянии h единиц друг от друга.

    Тогда наклон секущей с использованием формулы наклона равен

    Наклон секущей = [f(x 0 + h) — f(x 0 )] / (x 0 + h — x 0 ) = [f(x 0 + h) — f(x 0 )] / h

    Из приведенного выше рисунка видно, что если Q подходит очень близко к P (сделав h → 0) и сливается с P, то секущая становится касательной в P, т. е. наклон касательной в P можно получить, применив h → 0 к наклону секущей. Итак,

    Наклон касательной в точке P = limₕ → 0 [f(x 0 + h) — f(x 0 )] / h

    Мы знаем, что это не что иное, как производная от f(x) при x = x 0 (по предельному определению производной (или) из первых принципов). т. е.

    • Наклон касательной при P = f ‘(x 0 )

    Следовательно, наклон касательной есть не что иное, как производная функции в точке, где она проведена.

    Формула наклона касательной

    Наклон касательной к y = f(x) в точке (x 0 , Y 0 ) IS (DY/DX) (x 0 , Y 0 ) (OR) (F ‘(x)) (x ) (or) (x)) (x ) (or) (x)) (x ) (or) (x)) ) ) (or) (x)) ) (or). y 0 ) , где

    • f'(x) — производная функции f(x).
    • (f ‘(x)) (x 0 , y 0 ) — значение, полученное путем подстановки (x, y) = (x 0 , y 3) производная f'(x).

    Обратите внимание, что нам, возможно, придется использовать неявное дифференцирование, чтобы найти производную f ‘(x), если функция определена неявно.

    Уравнение касательной линии

    Мы знаем, что уравнение линии с наклоном ‘m’, проходящей через точку (x 0 , y 0 ), находится с использованием формы точка-наклон: y — y 0 = m (х — х 0 ). Рассмотрим касательную, проведенную к кривой y = f(x) в точке (x 0 , у 0 ). Тогда из предыдущих разделов

    Наклон касательной, м = (f'(x)) (x 0 , y 0 )

    2 , и значения y 0 в форме точка-наклон y — y 0 = m (x — x 0 ) мы можем получить уравнение касательной.

    Таким образом, формула касательной:

    • y — y 0 = (f ‘(x)) (x 0 , у 0 ) (х — х 0 )

    Шаги для нахождения уравнения касательной

    Чтобы найти уравнение касательной к кривой y = f(x), проведенной в точке (x 0 , y 0 ) (или в точке x = x 0 ):

    • Шаг — 1: Если координата y точки НЕ дана, т. е. если в вопросе говорится, что касательная проведена в точке x = x 0 , то найдите координату y, подставив ее в функции y = f(x).
      то есть координата y, y 0 = f(x 0 ).
    • Шаг — 2: Найдите производную функции y = f(x) и представьте ее в виде f'(x).
    • Шаг — 3: Подставьте точку (x 0 , y 0 ) в производную f ‘(x), которая дает наклон касательной (m).
    • Шаг — 4: Найдите уравнение касательной, используя форму точка-наклон y — y 0 = m (x — x 0 ).

    Приближение касательной линии

    Концепция линейной аппроксимации просто следует из уравнения касательной. т. е. уравнение касательной функции y = f(x) в точке (x 0 , y 0 ) можно использовать для аппроксимации значения функции в любой точке, очень близкой к ( х 0 , у 0 ). Мы можем понять это из примера ниже.

    Пример аппроксимации касательной линии

    Используйте аппроксимацию касательной, чтобы найти приблизительное значение ∛8.1.

    Решение

    Мы знаем, что ∛8 = 2 и 8,1 очень близко к 8.

    Итак, мы предполагаем, что функция равна f(x) = ∛x, а точка, в которой проведена касательная, равна x 0 = 8.

    Тогда (x 0 , y 0 ) = (8, ∛8) = (8, 2).

    Производная функции f'(x) = (1/3) x -2/3

    Наклон касательной равен, m = (f'(x))₍₈, ₂ ) = (1/3) (8) -2/3 = (1/3) (2 3 ) -2/3 = (1/3) (1/4) = 1/ 12

    Уравнение касательной: y — y 0 = m (x — x 0 )

    y — 2 = (1/12) (x — 8)

    y = x/12 — 2/3 + 2

    y = x/12 + 4/3

    Подставив здесь y = f(x),

    f(x) = x/12 + 4/3

    Теперь приблизительное значение ∛8.1 можно получить, подставив здесь x = 8.1. Таким образом,

    f(8. 1) ≈ (8.1)/12 + 4/3

    ∛8,1 ≈ 2,008

    Мы можем проверить это с помощью калькулятора, найдя кубический корень из 8,1, и мы увидим, что он равен 2,008. Вот как работает аппроксимация касательной.

    Уравнение касательной к параметрической кривой

    Иногда функция кривой может быть представлена ​​не в виде y = f(x), а в параметрической форме. Давайте посмотрим, как найти уравнение касательной к параметрической кривой как в 2D, так и в 3D.

    Касательная линия параметрической кривой в 2D

    Если кривая в 2D представлена ​​параметрическими уравнениями x = x(t) и y = y(t), то уравнение касательной линии при t = a находится с помощью следующие шаги:

    • Найдите точку, в которой проведена касательная (x 0 , y 0 ), подставив t = a в заданные параметрические уравнения.
      т. е. (x 0 , y 0 ) = (x(a), y(a)).
    • Найдите производную функции, используя (dy/dt) / (dx/dt).
    • Найдите наклон касательной (m), подставив либо t = a в приведенную выше производную.
    • Найдите уравнение касательной, используя y — y 0 = m (x — x 0 ).

    Касательная линия параметрической кривой в 3D

    Пусть кривая в 3D определяется параметрическими уравнениями x = x(t), y = y(t) и z = z(t). Вот шаги, чтобы найти уравнение касательной в точке t = t 0 .

    • Подставьте t = a в каждое из данных уравнений, чтобы найти точку (x 0 , y 0 , z 0 ), в которой проведена касательная.
      т. е. (x 0 , y 0 , z 0 ) = (x(t 0 ), y(t 0 ) и z(t 0 ))
    • Найдите производные x'(t), y'(t) и z'(t).
    • Подставьте t = t 0 в каждую из этих производных, чтобы найти отношения направлений линии.
      т. е. = 0 ), y'(t 0 ), z'(t 0 )>
    • Найдите уравнение касательной по одной из следующих формул:
      x = x 0 + at, y = y 0 + bt, z = z 0 + ct [OR]
      ( x — x 0 ) / a = (y — y 0 ) / b = (z — z 0 ) / c

    Мы можем увидеть примеры этих формул в разделе «Примеры» ниже.

    Касательная линия полярной кривой

    Если функция определяется полярным уравнением r = r(t), то уравнение касательной при t = a находится с помощью следующих шагов:

    • Найдите ‘r’, где r = r(a) .
    • Найдите точку (x 0 , y 0 ), в которой проведена касательная, используя (x 0 , y 0 ) = (r cos a, r sin a).
    • Найдите dy/dx по формуле \(\dfrac{d y}{d x}=\dfrac{\dfrac{d r}{d t} \sin (t)+r \cos (t)}{\dfrac{d r} {d t} \cos (t)-r \sin (t)}\).
    • Найдите наклон касательной, используя m = (dy/dx) t = a .
    • Найдите уравнение касательной, используя y — y 0 = m (x — x 0 ).

    Важные замечания по касательной:

    • Уравнение касательной кривой y = f(x) в точке (x 0 , y 0 ) находится с помощью y — y 4 0

      = м (х — х 0 ),
      где m = (f ‘(x)) (x 0, у 0 ) .

    • Если θ представляет собой угол, образуемый касательной с положительным направлением оси x, то ее наклон равен m = tan θ.
    • Линия нормали и касательная, проведенные для кривой в точке, перпендикулярны друг другу, поэтому наклон нормали = (-1) / (наклон касательной).
    • Кривая y = f(x) имеет горизонтальные касательные в точках, где f ‘(x) = 0, поскольку горизонтальные касательные параллельны оси x.
    • Кривая y = f(x) имеет вертикальные касательные в точках, где f ‘(x) не определена, поскольку горизонтальные касательные параллельны оси y.
    • Уравнение касательной используется для нахождения приблизительных значений функции в окрестности точки, в которой проведена касательная.

    ☛  Связанные темы:

    • Калькулятор касательной
    • Калькулятор производных
    • Расчетный калькулятор

    Часто задаваемые вопросы о касательной линии

    Что такое определение касательной линии?

    Касательная кривой y = f(x) — это линия, которая касается кривой в точке (x 0 , y 0 ). Его наклон (m) находится путем подстановки точки, в которой он начерчен, в производную f'(x), а его уравнение находится с использованием y — y 0 = m (x — x 0 ).

    Как найти наклон касательной?

    Наклон касательной в точке является ее производной в этой точке. Если провести касательную к кривой y = f(x) в точке (x 0 , y 0 ), то ее наклон (m) получается простой подстановкой точки в производную функции. т.е.

    Что означает точка касания?

    Касательная линия кривой касается кривой в одной точке, и эта точка называется точкой касания. Это очень важно при нахождении уравнения касательной.

    Как найти уравнение касательной для y = f(x)?

    Найти уравнение касательной прямой y = f(x) при x = x 0 :

    • Найти точку (x 0 , y 0 ) = (x 0 , f( х 0 )).
    • Найдите наклон, используя m, подставив (x, y) = (x 0 , y 0 ) в f'(x), где f'(x) — производная от f(x).
    • Найдите уравнение касательной, используя y — y 0 = m (x — x 0 ).

    Как найти уравнение касательной к параметрической кривой в 2D?

    Если кривая задана параметрическими уравнениями x = x(t) и y = y(t), то уравнение касательной линии, проведенной при t = a, находится с помощью следующих шагов:

    • Найдите точку (x 0 , у 0 ) = (х(а), у(а)).
    • Найдите наклон dy/dx, используя dy/dx = (dy/dt) / (dx/dt).
    • Найдите наклон касательной, используя m = (dy/dx) t = a
    • Найдите уравнение касательной, используя y — y 0 = m (x — x 0 ).

    Может ли касательная пересекать кривую?

    Единственным условием для того, чтобы линия была касательной кривой в точке, является то, что линия должна касаться кривой в этой точке. Но он может пересечь график в любой другой точке (точках).

    Как найти уравнение касательной к полярной кривой?

    Если кривая задана полярным уравнением r = r(t), то уравнение касательной линии, проведенной при t = a, находится с помощью следующих шагов:

    • Найдите r = r(a).
    • Найдите точку (x 0 , y 0 ) = (r cos a, r sin a).
    • Найдите наклон dy/dx, используя dy/dx = \(\dfrac{\dfrac{d r}{d t} \sin (t)+r \cos (t)}{\dfrac{d r}{d t} \cos ( т)-р \sin(t)}\)
    • Найдите наклон касательной, используя m = (dy/dx) t = a .
    • Найдите уравнение касательной, используя форму точка-наклон, y — y 0 = m (x — x 0 ).

    Как найти, где касательная вертикальна?

    Наклон вертикальной линии не определен. Кроме того, мы знаем, что наклон касательной равен производной. Таким образом, чтобы увидеть, где касательная вертикальна, просто посмотрите, где производная не определена. Если производная рациональна, просто установите знаменатель = 0 и решите.

    Как найти уравнение касательной к параметрической кривой в 3D?

    Если кривая в трех измерениях определяется параметрическими уравнениями x = x(t), y = y(t) и z = z(t), то уравнение касательной, проведенное к ней в точке x = t 0 находится следующим образом:

    • Найдите точку, используя г(т 0 ))
    • Найдите производные заданных функций. т. е. найти x'(t), y'(t) и z'(t)
    • Найдите направление линии, используя = 0 ), y'(t 0 ), z'(t 0 )>
    • Найдите уравнение касательной с помощью x = x 0 + at, y = y 0 + bt, z = z 0 + ct (параметрическая форма) или (x — x 0 ) / a = (y — y 0 ) / b = (z — z 0 ) / c (декартова форма)

    Как найти уравнение горизонтальной касательной?

    Горизонтальная касательная параллельна оси X, поэтому ее наклон равен нулю. Мы знаем, что наклон есть не что иное, как производная функции. Итак, чтобы найти точки, в которых есть горизонтальные касательные, просто установите производную функции равной нулю и решите. Получив точки, мы можем найти уравнение горизонтальной касательной, используя форму точка-наклон.

    Как найти уравнение вертикальной касательной?

    Вертикальная касательная параллельна оси Y, поэтому ее наклон не определен. Поскольку наклон — это не что иное, как производная функции, чтобы найти точки, в которых есть вертикальные касательные, посмотрите, где производная функции становится неопределенной (вероятно, установите знаменатель производной равным нулю, чтобы найти его). Получив точки, мы можем найти уравнение вертикальной касательной, используя форму точка-наклон.

    В чем разница между вертикальными и горизонтальными касательными?

    Наклон горизонтальной касательной равен 0 (т. е. производная равна 0), поскольку она параллельна оси x. Наклон вертикальной касательной не определен (знаменатель производной равен 0), поскольку он параллелен оси y.

    4.2: Наклон касательной

    1. Последнее обновление
    2. Сохранить как PDF
  • Идентификатор страницы
    1243
  • Кевин изучает основы исчисления и то, для чего на самом деле используется исчисление. К сожалению, Кевин не понимает, почему вычисления иногда необходимы, чтобы найти уравнение прямой. В Алгебре 1 он узнал, что можно найти уравнение прямой, если вам даны две точки. Вы находите наклон линии, разделив разницу между точками вверх/вниз на разницу влево/вправо, затем вы используете одну из точек и наклон, чтобы найти точку пересечения по оси y.

    Учитель Кевина, мистер Бэннер, предложил ему дополнительные баллы, если он сможет найти наклон линии для точек (4,5) и (4,5), используя метод, который он изучил на Алгебре 1. Баннер делал? Что обнаружит Кевин, работая над этими проблемами?


    ТАНГЕНТЫ К кривой

    Отзыв из алгебры, если точки P ( x 0 , Y , Y 0823082308230823082308230823.08230823082308230823082308230823082308230823082308230823082308230823082н.0401 ( x 1 , y 1 ) are two different points on the curve y = f ( x ), then the slope из Secant Line , соединяющая две точки, дается

    Конечно, если мы позволим точке x 1 . 0400 Q will approach P along the graph f and thus the slope of the secant line will gradually approach the slope of the tangent line as x 1 приближается к x 0 . Таким образом, (1) принимает вид

    . Для упрощения обозначений, если принять h = x 1 x 0 , затем x 1 = x 0 + 1 0 + H

    0 + 0001

    0 +

    . This means that (2) becomes

    Recall that the equation of the tangent line through point ( x 0 , y 0 ) with slope m is точечно-наклонная форма линии: Y Y 0 = M TAN ( x 1.

    ( x .

    ( x

    ).


    Примеры

    Пример 1

    Ранее вам дали задачу о Кевине, у которого проблемы с пониманием исчисления.

    Мистер Бэннер попросил Кевина найти уравнение прямой по точкам (4,5) и (4,5). Точки (4, 5) и (4, 5) совпадают, поэтому подняться / запустить будет 00 — Кевин только что узнал о необходимости дифференциального исчисления !

    Пример 2

    Найти линию Tangence to Crve F ( x ) = x 3 , который проходит по точке 3 .

    Поскольку P ( x 0 , y 0 )0003

    Таким образом, наклон касательной равен 12. Используя вышеприведенную формулу точки-наклона, мы находим уравнение касательной прямой: or y = 12 x — 16.

    Example 3

    If f ( x ) = x 2 − 3,find f’ ( х ) и используйте результат, чтобы найти наклон касательной в точках x = 2 и x = −1.

    Since then

    To find the slope, we simply substitute x = 2 into the result f’ ( x ):

    and

    Thus наклон касательной на x = 2 и x = -1 равно 4 и -2 соответственно.

    Пример 4

    Найдите наклон касательной к кривой y = 1/ x , проходящей через точку (1, 1).

    Using the slope of the tangent formula,

    Thus the slope of the tangent line at x = 1 for the curve y = 1/ x is m = -1. Чтобы найти уравнение касательной линии, мы просто используем формулу точки-наклона,

    Таким образом, уравнение касательной линии составляет y = — x + 2.

    Пример 5

    . значения x 0 =3 и x 1 =4, найти:

    1. 0 , x 1 ].

    Определите две точки, заменив 3 и 4 на x в функции f(x)= 1 / 2 x 2

    Подставьте две точки (3, 4,002) 4, 8) в формулу средней скорости изменения: m = y 1 −y 0 /x 1 −x 0

    1. Наклон секущей, соединяющей x 0 и x 1.

    Наклон секущей между x 0 и x 1 — это наклон между (3,4,5) и (4,8), равный 72.

    1. Мгновенная скорость изменения из y относительно x на x 0 .

    Мгновенная скорость изменения наклона at x = 3.

    Используйте формулу: f(x+h)−f(x) / h , где f(x)= 1 / 2 x

    0 2 и x=3

    f(3+h)−f(3) / h ….. Подставим 3 вместо x

    0,5(3+h)2−0,5(3)2 / H ….. Замените F (x) → 1 / 2 x 2

    Фольга и распределить 1 / 2

    / 2

    / 2

    / 2 совпадает с мгновенной скоростью изменения при x=4

    Это та же последовательность шагов, что и при x = 3 выше

    ∴ наклон при x = 4 равен 4

    Пример 6

    Учитывая функцию f(x)=1x и значения x 0 =2 и x 1 = 3, Найти:

    1. Средняя скорость смены y относительно x над интервалом [ x 0 [ x 0. 1010101010101010101010101010101010101010101010101010101010101010101040101010401.

      0. . 1 ].

    Определите две точки, подставив 2 и 3 вместо x в функции

    Подставим две точки (2, 1 / 2 )|(3, 1 / 3 ) в формулу средней скорости изменения 6 Y 1 -Y 0 /x 1 −x 0

    Средняя скорость изменения = −16

    1. Наклон из линии Secant Line, соединяющий 2
      1. . Наклон из линии Secant, соединяющий . x
        1. . Наклон из линии Secant Connecting

          1. . х 1 .

          Наклон секущей между x0 и x1 равен наклону между (2, 1 / 2 ) и (3, 1 / 3 ), который равен −16.

            1. Мгновенная скорость изменения y по отношению к x на x 0 .

            Мгновенная скорость изменения при x 0 наклон при x = 2.

            Используйте формулу: f(x+h)−f(x) / h где f(x)= 1 / x и x=2

            2

            2

            У нас была дробь, разделенная на дробь, инвертировать, чтобы умножить

            Наклон касательной при 3 такой же, как мгновенная скорость изменения при x=3

            Это та же последовательность шагов, что и при x = 2 выше

            ∴ наклон при x = 3 равен −1 / 9


            Обзор

            1. Как называется линия, соединяющая две точки (x 0 ,y 0 ) и (x 1 ,y 1 ) на кривой?
            2. Поскольку (x 0 ,y 0 ) становится неизмеримо близким к (x 1 ,y 1 ), термин, описывающий линию между ними, становится: «____________ линия»
            3. Выражение f(x 0 +h)−f(x 0 ) используется для описания какого расстояния в процессе нахождения наклона касательной?
            4. При вычислении наклона касательной какое значение предполагается равным 0 по мере сближения двух выбранных точек?
            5. Какое отношение понятие предела, обсуждавшееся в предыдущих уроках, имеет к нахождению наклона прямой, касательной к кривой?

            Найдите уравнение касательной:

            1. Каким будет уравнение касательной в точке x=−3, если предположить, что r(−3)=−5 и r′(−3)=1?
            2. Каково уравнение касательной в точке x=1, если предположить, что r(1)=3 и r′(1)=−5?
            3. Каково уравнение касательной в точке x=2, если предположить, что g(2)=1 и g′(2)=−3?
            4. Каково уравнение касательной в точке x=4, если предположить, что u(4)=4 и u′(4)=3?
            5. Каково уравнение касательной в точке x=−4, если предположить, что t(−4)=2 и t′(−4)=5?

            Найдите уравнение касательной:

            1. Найдите уравнение касательной к графику h(x)=−5x 3 −3x 2 +x+3 при x=1
            2. Найдите уравнение касательной к графику t(x)=−2x при x=−2
            3. Найти уравнение касательной к графику m(x)=3x 3 +3x 2 +4x+4 при x=1
            4. Найти уравнение касательной к графику q(x)=−x 3 −4x 2 +4x+3 при x=−2
            5. Найти уравнение касательной к графику t(x)=−4x 2 +2x−4 при x=−1
            6. Найдите уравнение касательной к графику h(x)=−4x 3 +2x 2 −3x+3 при x=−1
            7. Найдите уравнение касательной к графику m(x)=x при x=0
            8. Найти уравнение касательной к графику s(x)=−3x 2 −2x+3 при x=0
            9. Найдите уравнение касательной к графику c(x)=−3 при x=0
            10. Найти уравнение касательной к графику b(x)=−5x 4 +3x 3 −x 2 +5x−3 при x=−1

            Обзор (ответы)

            Чтобы просмотреть ответы в обзоре, откройте этот PDF-файл и найдите раздел 8. 7.


            Словарь

            Срок Определение
            секанс Линия, пересекающая окружность в двух точках.
            тангенс Линия, пересекающая окружность ровно в одной точке.
            Средняя скорость изменения Средняя скорость изменения функции — это отношение изменения координаты y функции к изменению координаты x.
            Дифференциальное исчисление Дифференциальное исчисление — это ветвь исчисления, основанная на нахождении разницы в расположении двух точек, которые сближаются до тех пор, пока расстояние между ними не станет бесконечно малым.
            мгновенная скорость изменения Мгновенная скорость изменения кривой в данной точке — это наклон линии, касательной к кривой в этой точке.
            секущая Секущая — это линия, соединяющая две точки на кривой.
            Наклон Наклон — это мера крутизны линии. Линия может иметь положительный, отрицательный, нулевой (горизонтальный) или неопределенный (вертикальный) наклон. Наклон линии можно найти, вычислив «подъем по отношению к пробегу» или «изменение y по сравнению с изменением x». Символ уклона м
            Касательная Касательная линия — это линия, которая «просто касается» кривой в одной точке и ни в какой другой.

            Дополнительные ресурсы

            PLIX: Play, Learn, Interact, eXplore — Slope of the Tangent and Secant Lines

            Видео: Уравнение касательной линии

            Практика: Наклон касательной линии

            Real World Off3 900: Кривая


            Эта страница под названием 4. 2: Slope of Tangent Line используется в соответствии с лицензией CK-12 и была создана, изменена и/или курирована Фондом CK-12 с использованием исходного контента, который был отредактирован в соответствии со стилем и стандартами платформы LibreTexts; подробная история редактирования доступна по запросу.

            ПОД ЛИЦЕНЗИЕЙ

            1. Наверх
              • Была ли эта статья полезной?
              1. Тип изделия
                Раздел или страница
                Автор
                СК12
                Лицензия
                СК-12
                Программа OER или Publisher
                СК-12
                Показать оглавление
                нет
              2. Теги
                1. средняя скорость изменения
                2. дифференциальное исчисление
                3. мгновенная скорость изменения
                4. секанс
                5. секущая линия
                6. склон
                7. источник@https://www. ck12.org/c/calculus
                8. тангенс
                9. касательная

              2.1 Наклон функции

              Предположим, что $y$ является функцией $x$, скажем, $y = f(x)$. Часто необходимо знать, насколько чувствительно значение $y$. заключается в небольших изменениях $x$.

              Один из способов интерпретировать приведенный выше расчет — это ссылка на линию. Мы вычислили наклон прямой через $(7,24)$ и $(7.1,23.9706)$, называемая хордой окружности. В общем случае, если провести хорду из точки $(7,24)$ в ближайшую точка на полуокружности $(7+\Delta x,\,f(7+\Delta x))$, наклон этой аккорд так называемый 92}-24\over \Дельта x}. $$ Например, если $x$ изменится только с 7 на 7,01, то коэффициент разности (наклон хорды) примерно равен $(23,997081-24)/0,01=-0,2919$. Это немного менее круто, чем хорда от $(7,24)$ до $(7,1,23,9706)$.

              Когда второе значение $7+\Delta x$ приближается к 7, соединение хорды $(7,f(7))$ до $(7+\Delta x,f(7+\Delta x))$ немного сдвигается. 2}}$, как и раньше. это НЕ всегда верно, что касательная перпендикулярна линии из начала координат — не используйте этот ярлык в любых других обстоятельствах.

              Как и выше, и, как вы могли ожидать, для различных значений $x$ мы обычно получают разные значения производной $f'(x)$. Может быть что производная всегда имеет одно и то же значение? Это будет означать, что наклон $f$, или наклон его касательной, одинаков везде. Одна кривая, которая всегда имеет один и тот же наклон, является линией; Это кажется странным говорить о касательной линии к прямой, но если это делает смысл вообще касательной линии должен быть самой линией. это не сложно убедиться, что производная от $f(x)=mx+b$ равна $f'(x)=m$; видеть упражнение 6. 92}$ между $x=0$ и $х=13$. Найдите наклон $\Delta y/\Delta x$ хорды между точки окружности, лежащие над (a) $x=12$ и $x=13$, (b) $x=12$ и $x=12,1$, (c) $x=12$ и $x=12,01$, (d) $x=12$ и $x=12,001$. Теперь используйте геометрия касательных линий на окружности, чтобы найти (e) точное значение производная $f'(12)$. 2}$ в тексте для каждого из следующих $x$: (a) 20, (б) 24, (в) $-7$, (г) $-15$. Нарисуйте график верхней полуокружности, и проведите касательную в каждой из этих четырех точек. (отвечать)

              Пример 2.1.3 Нарисуйте график функции $y=f(x)=1/x$ между $x=1/2$ и $x=4$. Найдите наклон хорды между (a) $x=3$ и $x=3,1$, (b) $x=3$ и $x=3,01$, (c) $x=3$ и $x=3,001$. Теперь используйте алгебру, чтобы найти простой формула наклона хорды между $(3,f(3))$ и $(3+\Delta х,f(3+\Дельта х))$. Определите, что происходит, когда $\Delta x$ приближается к 0. На графике $y=1/x$ проведите прямую через точку $(3,1/3)$, наклон которой представляет собой это предельное значение разностного отношения как $\Delta x$ приближается к 0. (отвечать) 93$ провести прямую через точка $(1,1)$, наклон которой равен только что найденному значению. (отвечать)

              Пример 2.1.6 Найдите алгебраическое выражение для разностного отношения $(f(x+\Delta x)-f(x))/\Delta x$, когда $f(x)=mx+b$. Упростите выражение как насколько это возможно. Затем определите, что происходит, когда $\Delta x$ приближается к 0. Это значение равно $f'(x)$. (отвечать)

              Пример 2.1.7 Нарисуйте единичный круг. Обсудите поведение склона касательной под разными углами по окружности. Который тригонометрическая функция дает наклон касательной под углом $\тета$? Почему? Подсказка: думайте в терминах отношений сторон треугольники. 92$. При каких значениях $x$ на положителен ли наклон касательной параболы? Отрицательный? Что вы заметили на графике в точках, где знак наклон изменяется с положительного на отрицательный и наоборот?

              Касательные линии: уравнение, наклон и график

              На латыни слово tangent означает «касаться». Итак, касательная — это линия, которая касается. Рассмотрим велосипед, движущийся по ровному тротуару. Дорога по существу касается велосипедного колеса, поскольку она касается колеса в одной точке. В этой статье мы дополнительно обсудим значение касательной, формулу касательной и значение наклона касательной.

              Определение и формула касательной линии

              Касательная линия — это линия, которая «просто касается» точки . Его также можно определить как линию, соединяющую две бесконечно близкие точки на кривой.

              Касательная к кривой в точке с координатами , — это линия, проходящая через с наклоном

              , если предел существует.

              Уравнение касательной

              Один наклон , уравнение касательной линии такое же, как и для любой другой линии в форме точка-наклон через точку:

              Касательные линии на графике

              На приведенном ниже графике мы говорим, что является касательной линия к кривой в точке . Или мы можем сказать, что касается кривой в точке .

              Касательная, выделенная зеленым цветом, просто касается кривой f в точке P — StudySmarter Original

              Обратите внимание, что касательная «просто касается» кривой в точке P.

              Наклон касательной

              Геометрия

              Наклон касательной в точке кривой равен наклону кривой в точке эта точка. Предположение, касающееся касательных линий, заключается в том, что при просмотре графика кривой, если вы увеличите масштаб достаточно близко к сегменту кривой, кривая будет выглядеть неотличимой от касательной.

              Например, давайте увеличим график выше.

              Увеличение в точке, где касательная линия касается кривой — StudySmarter Original

              Увеличение еще немного…

              Здесь мы видим, что касательная и точка кривой, где касается касательная, неразличимы — StudySmarter Original

              Обратите внимание на то, как линия касательной получается из соединения двух бесконечно близких точек на кривой.

              Дополнительное уравнение наклона касательной

              Существует еще одна версия уравнения для наклона касательной, с которой может быть проще работать. Это уравнение говорит о том, что наклон касательной равно

              Это уравнение устанавливает и . Поскольку приближается к , приближается к 0. Таким образом, образуется дополнительное уравнение касательной.

              Это уравнение для наклона касательной должно показаться вам знакомым… Это уравнение для производной функции в точке (a, f(a)). Итак, можно сказать, что наклон касательной к кривой в точке P равен производной кривой в точке Р !

              Примеры нахождения уравнения касательной

              Пример 1

              Найдите уравнение для касательной к в точке (2, 4).

              Поскольку нам дана точка, все, что нам нужно, чтобы составить уравнение касательной, это наклон. Чтобы найти наклон, воспользуемся дополнительным уравнением касательной.

              Наклон касательной в точках (2, 4) равен

              Итак, уравнение касательной к f(x) по адресу (2, 4) есть.

              Вспомните, как наклон касательной совпадает с производной. Таким образом, мы также можем просто взять производную и подставить ее, чтобы найти наклон касательной в точке .

              График функции f(x) и касательной к f(x) в точках (2, 4) — StudySmarter Original

              Пример 2

              точка (1, 0).

              Опять же, поскольку нам дана точка, все, что нам нужно, чтобы составить уравнение касательной, это наклон. Чтобы найти наклон, воспользуемся дополнительным уравнением касательной.

              При , наклон касательной в точке (1, 0) равен:

              Итак, уравнение касательной к f(x) в точке (1, 0) равно .

              Опять же, вспомним, что наклон касательной совпадает с производной. Таким образом, мы также можем просто взять производную и подставить 1, чтобы найти наклон касательной в точке .

              График f(x) и касательная к f(x) в точке (1, 0) — StudySmarter Original

              Касательные линии в окружности

              Говорят, что прямая касается окружности , если она касается окружности ровно в одной точке. Если линия касается окружности в точке, то касательная линия перпендикулярна радиусу, проведенному к точке.

      Добавить комментарий

      Ваш адрес email не будет опубликован. Обязательные поля помечены *